Вы находитесь на странице: 1из 37

DIAGNÓSTICO Los antidepresivos tricíclicos se utilizan en el tratamiento de la pirosis

funcional por desensibilizar el esófago.


CLÍNICO CASO 1
CLÍNICO CASO 2
Mujer de 32 años de edad, acude a consulta por pirosis
Hombre de 60 años de edad con antecedente de
diaria. Está en tratamiento con pantoprazol 40 mg 30
tabaquismo con IT de 25 e ingesta abundante de
min antes del desayuno desde hace 4 semanas y no ha
alcohol (40 gramos al día). Inició hace 8 semanas con
notado mejoría clínica. Niega náuseas, vómito,
evacuaciones diarreicas en cantidad aproximada de 5
disfagia y pérdida de peso.
a 6 al día, con esteatorrea y lientería. Las evacuaciones
La conducta a seguir es:
disminuyen con el ayuno y casi no se levanta durante
Solicitar panendoscopia
la noche al baño a menos que su último alimento haya
Incrementar dosis de Inhibidores de la bomba de
sido copioso. Durante este período ha perdido
protones
aproximadamente 3 kg de peso.
Solicitar pHmetría
De acuerdo a sus características clínicas ¿Cómo
Agregar ranitidina
La pirosis es uno de los síntomas cardinales de Enfermedad por reflujo
clasificaría la diarrea?
gastroesofágico. Está indicado realizar estudios de extensión cuando no hay Acuosa
una respuesta adecuada a IBP con dosis doble. En este caso la paciente tiene Malabsortiva
monodosis por lo que no es posible catalogarlo como ERGE refractario.
Tampoco tiene datos de alarma. Inflamatoria
Después de haber tomado pantoprazol dos veces al Funcional
Se trata de una diarrea malabsortiva (tiene esteatorrea y lientería), mejora
día, continúa con misma sintomatología. Usted decide: con el ayuno y empeora con la alimentación.
Solicitar pHmetría ¿Cuál de los siguientes no sería un diagnóstico
Solicitar panendoscopia diferencial en este caso?
Realizar esofagograma Atrofia de vellosidades intestinales
Agregar antiácidos Deficiencia de lactasa
En ERGE refractario está indicado realizar panendoscopia para descartar
otras causas. Sobrepoblación bacteriana
El resultado del estudio fue normal. Con lo anterior Insuficiencia pancreática exocrina
Todas las opciones comentadas ocasionan diarrea malabsortiva ya sea
usted decide: originada en la pared intestinal o en el lumen intestinal. La deficiencia de
Solicitar pHmetría lactasa no tiene por qué cursar con malabsorción de grasas y ocasionar
Solicitar manometría esofágica esteatorrea.

Solicitar esofagograma Dentro del abordaje se realizó una tomografía de


Enviar a cirugía abdomen que mostró calcificaciones difusas en el
No tiene evidencia de reflujo erosivo, por lo que es necesario hacer pHmetría parénquima pancreático y dilatación del conducto
para ver si existe reflujo patológico y descartar otras patologías como pirosis pancreático hasta 5 mm. ¿Cuál es la causa más
funcional.
frecuente de este padecimiento?
El estudio fue reportado como normal. El diagnóstico
Consumo de alcohol
más probable en esta paciente es:
Aumento en la cantidad de bacterias no patógenas en el
Enfermedad por reflujo gastroesofágico refractaria a
intestino delgado
tratamiento
Cambios en el microbioma intestinal
Pirosis funcional
Reacción alérgica al gluten
Trastorno psiquiátrico La causa más frecuente de pancreatitis crónica calcificante es el consumo de
Ninguna de las anteriores alcohol.
¿Cuál de los siguientes tratamientos es apropiado para ¿Cómo esperaría encontrar el resultado de la prueba
esta paciente? de D-xilosa y por qué motivo?
Omeprazol a altas dosis Normal – se requiere pérdida del 90% de la función
Antidepresivo tricíclico exocrina del páncreas para alterar este resultado
Ranitidina nocturna y omeprazol Anormal – Con alteraciones sutiles de la función
Antiácidos pancreática este resultado se altera
Normal – La D-xilosa requiere únicamente una pared individualizado, debe evitarse un abordaje extenso con múltiples estudios
innecesarios. Esta paciente no tiene datos de alarma y no hay algo que
intestinal íntegra para su absorción oriente que requiera estudios invasivos para su abordaje. El siguiente paso
Normal – La D-xilosa se absorbe en el colon para el abordaje diagnóstico en una constipación primaria es la manometría
En este caso la diarrea parece por pancreatitis crónica (insuficiencia anorrectal.
pancreática exocrina). La D-xilosa es un monosacárido que únicamente El estudio previamente indicado por usted se reportó
requiere integridad de la pared intestinal para su absorción. En pacientes con
insuficiencia pancreática y una pared intestinal íntegra, este resultado no
normal. ¿Cuál es el siguiente paso en el abordaje
tiene por qué alterarse. diagnóstico?
¿Cuál es el tratamiento de elección en este caso? Rectosigmoidoscopia
Tetraciclinas por 6 meses Enema baritado
Enzimas pancreáticas Tránsito colónico
Colestiramina Tránsito intestinal
Descompresión endoscópica del conducto pancreático Una vez que se descartaron trastornos defecatorios mediante manometría
El tratamiento de elección para la insuficiencia pancreática exocrina son las ano-rectal. El siguiente paso en el abordaje de la constipación primaria o
enzimas pancreáticas. Los tratamientos derivativos tanto quirúrgico como idiopática, es medir el tránsito colónico. Esto se logra mediante marcadores
endoscópico en pancreatitis crónica, se utilizan para el tratamiento del dolor. radio-opacos que se ingieren en cápsulas. Existen múltiples protocolos
validados para clasificar si el tránsito colónico normal o es lento.
¿Cuál de los siguientes tratamientos de constipación
CLÍNICO CASO 3
ocasiona melanosis coli?
Mujer de 24 años de edad sin antecedentes de
Senósidos
relevancia. Inició hace 4 años con episodios de
Polietilenglicol
constipación caracterizada por heces duras y en
Lactulosa
escíbalos acompañadas de dificultad para evacuar, en
Psyllium Plantago
ocasiones acompañadas de tenesmo. Niega otros Los laxantes estimulantes se asocian a melanosis coli y colon catártico. En
síntomas. algunos casos, el hallazgo colonoscópico de melanosis coli orienta al abuso
¿Cuál diagnóstico diferencial es el menos probable? de laxantes estimulantes.

Constipación de tránsito colónico lento


Trastorno defecatorio con disinergia CLÍNICO CASO 4
Constipación funcional Mujer de 75 años de edad diabética de larga evolución
Síndrome de intestino irritable variedad constipación sin complicaciones crónicas, con arritmia cardíaca no
Para integrar el diagnóstico de síndrome de intestino irritable es necesario especificada que nunca le ha condicionado síntomas.
que el paciente tenga dolor abdominal y en este caso no hay dolor Inició hace 2 horas con dolor abdominal mesogástrico
abdominal. El resto de las opciones son formas de constipación primaria o
idiopática y son diagnósticos diferenciales en este caso. Se requeriría mayor
intenso, acompañado de náusea y vómito. A la
información para inclinarse por algún diagnóstico específico. exploración física se encuentra intranquila,
¿Cuál es el tratamiento de elección inicial en este caso? diaforética, con facies álgica. Taquicárdica con FC de
Polietilenglicol 100, se escucha ritmo cardíaco irregular, FR 20, TA
Fibra 140/80mmHg, temperatura de 36 °C. La exploración
Lactulosa abdominal destaca solo por disminución discreta de los
Antiespasmódicos ruidos peristálticos en intensidad y frecuencia. No hay
El tratamiento inicial de la constipación es fibra seguida por laxantes de datos de irritación peritoneal. Se solicitó
preferencia osmóticos.
electrocardiograma que mostró ritmo “irregularmente
En caso de que no mejorara con el tratamiento
irregular” con ausencia de ondas P. La radiografía de
indicado y de acuerdo a las características clínicas.
abdomen no mostró alteraciones.
¿Cuál es el abordaje diagnóstico a seguir en este caso?
¿Cuál de los siguientes sería su primer estudio
Colonoscopia
diagnóstico?
Manometría anorrectal
Ultrasonido abdominal
Rectosigmoidoscopia
Resonancia magnética de abdomen
Enema baritado
El primer paso en el abordaje de constipación es un buen interrogatorio y Tomografía de abdomen contrastada
exploración para descartar causas secundarias. El abordaje debe ser Tomografía de abdomen no contrastada
Es un caso probable de isquemia mesentérica donde los hallazgos a la EF son CLINICAL CASE 5
desproporcionales a la intensidad del dolor del paciente. El primer paso en el
abordaje diagnóstico es la TAC contrastada de abdomen o si es posible y está A 50-year-old man with diabetes mellitus presents
al alcance una angiotomografía de abdomen. with melena and microcytic anemia. He reports having
El estudio solicitado mostró un defecto de llenado en epigastric pain in the last week. His only medication at
la arteria mesentérica superior. No se observa gas this moment is metformin and insulin. Upon admission
venoso portal ni neumatosis intestinal. ¿Cuál de las to the hospital his BP is 90/50 mmHg and HR 110 beats
siguientes es la etiología más probable en esta per minute. His hemoglobin is 8.5 mg/dL and INR 1.3.
paciente? What is the most appropriate step in the management
Trombosis arterial mesentérica in situ of this patient?
Embolismo a la arteria mesentérica superior Nasogastric lavage to evaluate for upper GI blood loss
Trombosis venosa mesentérica Administration of a proton pump inhibitor
Variante anatómica congénita Administration of intravenous fluids
De acuerdo al caso presentado la paciente parece tener fibrilación auricular. Urgent endoscopy
Esta entidad particularmente en pacientes de edad avanzada, con diabetes y
La administración de IBP forma parte del manejo en sangrado de tubo
con otros factores de riesgo que se encuentran en el acrónimo CHADS-VASC,
digestivo alto, sin embargo el paciente tiene datos que sugieren hipovolemia
tienen riesgo de trombosis en la orejuela del atrio izquierdo con ulterior
(hipotensión y taquicardia), por lo que el manejo inicial en este caso es
embolismo. El riesgo trombótico incrementa a mayor número de factores de
reanimación con líquidos intravenosos.
riesgo.
¿Cuál es el sitio anatómico más frecuentemente After 2 hours, his vital signs are stable. What is the next
afectado en estos casos? step?
Ostium de la arteria mesentérica superior Transfusion of fresh frozen plasma
Ostium del tronco celíaco Prophylactic administration of antibiotics
Arteria mesentérica superior distal a arteria cólica Hospital discharge with PPI therapy
media Upper gastrointestinal endoscopy
De acuerdo con sus estudios de laboratorios el paciente no tiene
Ostium de la arteria mesentérica requerimientos transfusionales. El paciente amerita estudio de endoscopia
Un 85% de los émbolos se aloja distal a la arteria cólica media. para localizar el sitio de sangrado.
¿Cuál de los siguientes es un sitio intestinal de An upper endoscopy reveals a clean based ulcer in the
susceptibilidad a la isquemia? duodenum. According to the result, which of the
Ángulo hepático following is the most likely etiology?
Ángulo esplénico Surreptitious use of non-steroidal anti-inflammatory
Ciego drugs
Íleon terminal Helicobacter pylori infection
El ángulo esplénico (Griffith) y la unión recto-sigmoidea (Sudeck), son sitios Malignant peptic ulcer
de susceptibilidad a la isquemia debido a que son áreas que comparten
circulación de arteria mesentérica superior con inferior. Zollinger-Ellison syndrome
¿Cuál es el paso siguiente en el tratamiento de la La causa más frecuente de úlcera duodenal es infección por H pylori (hasta
90% de los casos)
paciente? Gastric biopsies were taken and the histopathology
Anticoagulación y LAPE demonstrated the presence of Helicobacter pylori.
Anticoagulación y angiografía Then you decide:
Esplenoportografía Dual therapy: PPI plus sucralfate
Laparotomía exploratoria Continue PPI therapy for four weeks
La paciente no tiene datos de indicación quirúrgica urgente como son datos
de irritación peritoneal y los estudios de imagen no muestran gas venoso Empiric therapy for Helicobacter pylori
portal ni neumatosis intestinal. Además se encuentra descrito que el tiempo Culture-guided therapy for H. pylori
de viabilidad y de ventana terapéutica en las isquemias mesentéricas Amerita tratamiento con IBP durante 8 semanas para lograr la curación
arteriales agudas es de aproximadamente 12 horas. completa de la úlcera. Toda úlcera asociada a H. pylori amerita tratamiento
de erradicación. Si es el primer esquema se opta por tratamiento empírico.
El tratamiento guiado con cultivo está indicado en falla al tratamiento.
The pathogenesis of peptic ulcer involves an imbalance Which is the most common cause of the patient clinical
between defense and aggravating factors. Which of picture?
the following can both damage the gastric mucosal Postoperative ileus
barrier and stimulate gastric acid secretion? Postoperative intestinal adhesions
Gastric Hypokalemia
Mucous Medications
Helicobacter pylori The most common cause of mechanical obstruction is postoperative
adhesions.
Non-steroidal anti-inflammatory drug
H. pylori produce amonio, el cual produce daño directo a la mucosa gástrica.
The patient had initial improvement with your
Por otro lado puede incrementar de forma transitoria la secreción de HCl por management with further deterioration after 48
hiperplasia de las células G. hours. What’s the best next step in the management?
Laparotomy
CLINICAL CASE 6 Colonoscopy decompression
A 60 year old woman with a history of hysterectomy Upper GI endoscopy
because of uterine leiomyoma’s, presents to the Continue with established management until 5 days
emergency department. She reports severe abdominal have elapsed
pain with biliary vomiting for the last 2 hours. Physical After 48 to 72 hours of no improvement with conservative management, the
examination reveals a woman in acute distress, her next step in the management of a mechanical bowel obstruction is surgery.

vital signs are normal, except for mild tachycardia (90


bpm). Abdomen examination reveals increased bowel CLINICAL CASE 7
sounds with a metallic pitch. There are no peritoneal A 25 year old woman presents to your office with a 3
signs. month history of 2-3 bloody bowel movements per day
What’s the best next step in the diagnostic approach? associated with tenesmus. Physical examination is
Abdominal USG unremarkable. You decide to order stool cultures and
Abdominal RX ova / parasites, getting with negative results.
Abdominal CT Which of the following is the best next step in the
Laparotomy diagnostic approach?
The first step in the diagnostic approach of a bowel obstruction is abdominal Watch and wait
RX. Colonoscopy
Which of the options would be discordant with the Abdominal CT
clinical picture? Abdominal MRI
Presence of air in the transverse colon The initial step in the approach of a chronic inflammatory diarrhea as in this
Absence of air in the colon patient is colonoscopy or at least rectosigmodoscopy.

Presence of air fluid levels The diagnostic test that you ordered ruled out tumors
Presence of a transition zone and vascular causes in the patient’s clinical picture.
This clinical picture is typical of mechanical acute bowel obstruction (history According to this, what’s your first differential
of abdominal surgery, increased bowel sounds with metallic pitch). It’s not diagnosis at this moment?
usual to find colonic gas in mechanical obstructions in contrast to paralytic
ileus. Anal fissure
Which of the next options is not appropriate in the Internal hemorrhoids
management of this patient? Ulcerative colitis
Nasogastric tube Intestinal vasculitis
The clinical picture is typical of ulcerative colitis. After ruling out important
High dose opiates causes of lower GI bleeding in a patient with this features, UC is one of our
Hydration first differential diagnosis.
Abdominal CT
Opiates may contribute to further obstruction because their effects in GI
motility.
Which of the following describes the most probable Abdominal USG revealed a nodular liver. You decide to
findings in the study ordered? order a Fibro scan with reveals fibrosis F4. Which of the
Flask shaped ulcers scattered throughout the colon following is not appropriate in the management of this
Erythema, friability and ulcers predominantly at the patient?
transverse colon Lower GI endoscopy
Erythema friability and ulcers starting in the rectum and Upper GI endoscopy
expanding towards proximal colon Periodic USG (every 6 months)
Complex perianal fistulae Immunization for HAV, HBV
The endoscopic features of UC are those listed and highlighted. UC affects Lower GI endoscopy has no role in this patient. He has no history of lower GI
exclusively the colonic epithelium. Crohn’s disease affects the whole bleeding and there are no other indications for this test at this moment.
intestinal wall. One of the studies revealed varices “at some place of
In mild forms of this disease, what’s the first choice his gut”. He doesn’t remember where. He remember
treatment? that the report revealed presence of red-spots among
Infliximab big varices. The patient does not have a history of GI
Mesalamine bleeding. Which of the following options is appropriate
Steroids at this point for the management?
Methotrexate Variceal band ligation + propranolol
Amino salicylates in their oral or topical forms are useful in all forms of UC.
In mild forms of the disease, sometimes is enough treatment with this drug Propranolol only
class to control disease activity. Wait until further decompensation occurs to intervene
Which of the following conditions is not associated Metoprolol only
with this disease? There are two choices for primary prophylaxis of variceal bleeding, both with
similar efficacy: non-selective beta blockers or variceal band ligation. But at
Pyoderma gangrenous this point, there’s no role for combination therapy in primary prophylaxis
Uveitis (Variceal band ligation + beta blocker). Although sclerotherapy was used for
Primary sclerosing cholangitis this purpose, we have more effective and safer options.

Duodenal cancer What’s the main imaging feature of primary hepatic


Duodenal cancer is not associated with ulcerative colitis. A colonic condition malignancies in patient with cirrhosis?
associated with duodenal cancer is familiar adenomatous polyposis. Hypo dense lesion without contrast enhancement
Enhancement in arterial phase with washout in latter
CLINICAL CASE 8 phases
A 31 year old man with long-standing HCV infection Enhancement in portal venous phase
presents to your office with progressive fatigue and Ring enhancement
lower extremity edema. Physical examination reveals Hepatocellular carcinoma in cirrhotic patients, can be diagnosed without
mild edema around the heels. His fibrosis status is biopsy, as long as the imaging features are typical as highlighted in the above
option.
unknown.
In patients with cirrhotic ascites, which of the
Apart from general laboratory tests, which of the
following options is not appropriate?
following is the next first step in the diagnostic
Paracentesis at first presentation
approach?
Paracentesis in hospitalized patients with ascites
Liver biopsy
Paracentesis only if there are clinical clues of infection
Abdominal USG
Large volume paracentesis with IV albumin form
Abdominal CT
patients with tense ascites
Upper GI endoscopy In general the threshold to obtain a sample from ascites in patient with
The first step in the approach to this patient is USG. USG is a non-invasive and cirrhosis is low. There are many cases of spontaneous bacterial peritonitis
inexpensive way to know the main features of liver anatomy, liver borders, without clinical features of infection.
its blood vessels and the presence or absence of splenomegaly. Right now,
fibro scan and other non-invasive methods for measuring liver fibrosis are
becoming more frequently used for this purpose. Even though liver biopsy CLINICAL CASE 9
remains as the gold standard for diagnosis of liver fibrosis it’s invasive, has 40 year old man with intermittent dysphagia to both
significant costs and complications, so is not commonly used as a first test.
solids and liquids, and retrosternal pain, with no
regurgitation. A cardiologist discarded ischemic Chief cells of the stomach
cardiomyopathy. Principal cells of the stomach
In this case you should suspect: El pepsinógeno es producido en el estómago por las células principales de las
glándulas oxínticas localizadas en el fondo gástrico y cuerpo gástrico.
Mechanic disorder
Which of the following is likely consequence of ileal
Motor disorder
resection?
Stenosis
Achalasia
Psychiatric disease
La disfagia intermitente y a líquidos orienta hacia causas motoras.
Vitamin B12 deficiency
What kind of dysphagia does this patient have? Atrophic gastritis
Esophageal Peptic ulcer
La vitamina B12 es absorbida a nivel del íleon distal. Por lo tanto, cualquier
Oropharyngeal patología que afecte el íleon distal (resección quirúrgica, enfermedad de
Mechanic Crohn) puede cursar deficiencia de esta vitamina.
Neurologic Which of the following structures undergoes receptive
La disfagia orofaríngea usualmente se acompaña de tos, aspiración o relaxation when a bolus of food is swallowed?
dificultad para la propulsión del bolo alimenticio, por lo tanto este paciente
tiene disfagia esofágica.
Pharynx
Which of these options is not routinely employed Upper esophageal sphincter
during dysphagia approach? Esophagus
Manometry Stomach
La relajación receptiva permite que el estómago acomode un mayor
Upper endoscopy contenido alimenticio dentro.
Abdominal tomography Which of the following hormones is released by the
Upper GI series presence of fat and proteins in the duodenum and has
La TAC no tiene papel en el abordaje de la disfagia. Sólo en pacientes con
neoplasias diagnosticadas tiene utilidad en el estadiaje.
a major effect to decrease gastric emptying?
A manometry showed contractions of 180mmHg, with Cholecystokinin (CCK)
simultaneous contractions in 10% of wet swallows and Gastrin
lower esophageal sphincter pressure of 20mmHg with Secretin
residual pressure of 5mmHg. The probable diagnosis is: Motilin
CCK y secretina son hormonas secretadas en el duodeno por las células I y S
Hypertensive lower esophageal sphincter pressure en respectivamente. Las proteínas y grasas estimulan su liberación, pero la
Nutcracker esophagus CCK es la única que puede retrasar el vaciamiento gástrico.
Achalasia Various proteolytic enzymes are secreted in an inactive
Cricopharyngeal achalasia form into the small intestine. Which of these are
Esófago en cascanueces tiene contracciones enérgicas con >180 mmHg, dolor important for activating one or more proteolytic
y disfagia.
enzymes, converting them to an active form?
First line treatment for the upper condition includes
Enterocinase
the following:
Trypsinogen and trypsin
PPI therapy and calcium channel blockers
Trypsin and enterocinase
Butylhyoscine
Trypsin, enterocinase and pepsin
Myotomy La enterocinasa activa de forma inicial al tripsinógeno (proenzima) para
Cricopharyngeal myotomy producir tripsina la cual puede activar otras moléculas de tripsinógeno.
Tratamiento con IBP y relajantes de musculo liso. Miotomía en casos (Fenómeno de cascada)
extremos.
Fisiología gastrointestinal / Disfagia y trastornos de
OTHER QUESTIONS motilidad esofágica / Enfermedad por Reflujo
The proenzyme pepsinogen is secreted mainly from Gastroesofágico
which of the following structures: CASO CLÍNICO 1
I cells of the duodenum Mujer de 75 años de edad con diagnóstico de artritis
G cells of the stomach reumatoide, consume naproxeno de forma regular
para control del dolor. Hace una semana presentó Con estos hallazgos el diagnóstico es:
melena. Usted decide realizar una panendoscopia en Anemia perniciosa
la que se observó una úlcera gástrica recubierta de Gastritis atrófica ambiental
fibrina. Gastritis crónica tipo B
¿Cuál es la localización más frecuente? Gastritis crónica tipo AB
Curvatura mayor La deficiencia de Vitamina B12, gastritis atrófica con anticuerpos positivos
hace el diagnóstico de anemia perniciosa.
Fondo
¿Cuál es la distribución típica en este tipo de gastritis
Antro
atrófica?
Incisura
Sitio más frecuentes son antro y curvatura menor
Fondo y cuerpo
De acuerdo con la clasificación de Forrest es: Antro
Forrest Ib Curvatura menor
Forrest IIa Cardias
Gastritis atrófica tipo A es la que se asocia con anemia perniciosa, y se localiza
Forrest IIc en cuerpo y fondo.
Forrest III ¿Qué alteración espera encontrar en los estudios de
Ia y Ib son con sangrado activo. IIa y IIb son con vaso visible y coágulo
adherido. Ulcera recubierta de fibrina o base limpia es Forrest III.
laboratorio?
El tratamiento de esta úlcera es el siguiente: Anemia microcítica
Vigilancia Anemia macrocítica
Inhibidor de la bomba de protones (IBP) Deficiencia de ácido fólico
Inyección con adrenalina Anemia por deficiencia de hierro
Anemia macrocítica (VCM elevado) por la deficiencia de vitamina B12 es lo
Sonda bipolar más característico.
Solo IBP, por ser Forrest III no amerita tratamiento endoscópico.
¿Cuál es el tratamiento de esta patología?
¿Cuál sería el manejo a largo plazo en esta paciente?
Hierro
Vigilancia
Cianocobalamina
Suspender naproxeno
Piridoxina
Naproxeno + IBP
Tiamina
Cambiar a otro antiinflamatorio no esteroideo Tratamiento es la reposición de vitamina B12 o cianocobalamina.
Es necesario suspender el agente causal que en este caso son los AINES.
Esta patología se ha asociado con:
¿Qué porcentaje de las úlceras gástricas son asociadas
Metaplasia intestinal
a H. pylori?
Tumor epidermoide
90%
Úlcera péptica
80%
Ulcera intestinal
60% Gastritis crónica predispone a metaplasia intestinal y displasia.
30%
El 60% de las ulceras gástricas se asocian a H pylori y el 90% de las duodenales
CASO CLÍNICO 3
Mujer de 20 años de edad quien desde hace 3 meses
CASO CLÍNICO 2
presenta dolor abdominal tipo ardoroso en epigastrio.
Mujer de 35 años de edad con anemia por deficiencia
Ha tomado un inhibidor de la bomba de protones
de vitamina B12, se le solicitaron anticuerpos anti-
durante 8 semanas sin respuesta. Niega datos de
células parietales que resultaron positivos. Se le realizó
alarma.
panendoscopia en la que se observó atrofia de la
mucosa gástrica. Las biopsias mostraron gastritis
crónica atrófica sin displasia.
El cuadro clínico es compatible con: Esofágica
Dispepsia Mecánica
Gastritis El cuadro clínico es compatible con acalasia que es un trastorno motor
esofágico.
Neoplasia gástrica
De los siguientes estudios, ¿cuál no forma parte del
Síndrome de intestino irritable
abordaje inicial en un paciente con disfagia?
Usted decide:
Tomografía abdominal
Realizar tomografía de abdomen
Panendoscopia
Solicitar prueba de aliento
Serie esofagogastroduodenal
Realizar panendoscopia
Manometría
Iniciar antidepresivo
Toda dispepsia no investigada amerita prueba de aliento para detección de
Se le realizó una serie esofagogastroduodenal
H. pylori. observando dilatación importante del esófago y signo
Los siguientes, uno no es un dato de alarma en esta del pico de pájaro. La manometría esofágica mostró
patología: falta de relajación del esfínter esofágico inferior y
Historia familiar de cáncer gástrico aperistalsis del cuerpo esofágico. ¿Cuál es el
Anemia diagnóstico de este paciente?
Edad menor a 45 años Esófago hipercontráctil
Vómito Obstrucción al flujo de salida
Datos de alarma asociados a dispepsia son indicación de panendoscopia Acalasia
como estudio inicial. Estos son historia familiar de cáncer gástrico, anemia,
vómito, edad mayor a 55 años, pérdida de peso. Espasmo esofágico difuso
El patrón manométrico característico de la acalasia es ausencia de relajación
¿Cuál es la etiología más frecuente de esta patología? del EEI y aperistalsis.
Fármacos De acuerdo con lo anterior, usted decide:
Úlcera péptica Enviar a cirugía
Cáncer gástrico Indicar bloqueadores de canales de calcio
Funcional Enviar para dilatación endoscópica
El 70% son funcionales. El 30% tienen etiología orgánica.
Realizar una panendoscopia
Tiene prueba de aliento negativa y panendoscopia Aunque el esofagograma y la manometría son diagnósticos, es necesario
normal. De acuerdo con estos resultados, ¿cuál sería el realizar panendoscopia para descartar pseudoacalasia.
tratamiento de elección? La panendoscopia se reportó con esófago dilatado y
Continuar IBP restos alimenticios. El tratamiento de elección en esta
Tratamiento empírico para H. pylori patología es:
Sucralfato Miotomía de Heller
Antidepresivo Toxina botulínica
El diagnóstico de la paciente es de dispepsia funcional, el tratamiento de Calcio antagonista
elección en dispepsia funcional es antidepresivo.
Inhibidores de fosfodiesterasa 5
Tratamiento de elección es con miotomía de Heller o dilatación endoscópica.
CASO CLÍNICO 4
Mujer de 35 años de edad con disfagia a sólidos y CASO CLÍNICO 5
líquidos de 6 meses de evolución, regurgitación, dolor Hombre de 75 años de edad con disfagia de 5 meses
retroesternal y pérdida de peso. de evolución al iniciar la deglución. Refiere sensación
Por las características de los síntomas, la disfagia es: de atoramiento a nivel del cuello, por lo que toma
Orofaríngea abundantes líquidos durante las comidas presentando
Faríngea regurgitación intermitente y halitosis.
¿Cuál es el diagnóstico más probable? meses de evolución de predominio a sólidos la cual ha
Disfagia orofaríngea sido progresiva.
Disfagia esofágica ¿Qué tipo de disfagia presenta la paciente?
Enfermedad por reflujo gastroesofágico Disfagia motora
Esófago hipercontráctil Disfagia orofaríngea
La disfagia orofaríngea implica dificultad para iniciar la deglución, la refieren Disfagia mecánica
como sensación de atoramiento en el cuello. Suele acompañarse de tos,
regurgitación y halitosis. Disfasia fisiológica
La disfagia que inicia con sólidos es por un problema mecánico
De acuerdo con su sospecha clínica, ¿qué estudio principalmente estenosis péptica. La disfagia que inicia con sólidos y líquidos
solicitaría de forma inicial? indica problema motor.
Panendoscopia ¿Qué estudio sería de utilidad en el abordaje de esta
pHmetría de 24 horas paciente?
Manometría esofágica Endoscopia
Videofluoroscopia Panendoscopia
En disfagia orofaríngea el estudio inicial es videofluoroscopia. El resto es para Manometría esofágica
disfagia esofágica.
Videofluoroscopia
Un trago de bario demostró la presencia de un Trago con bario y panendoscopia son de utilidad en disfagia mecánica. La
divertículo en el tercio superior del esófago. Éste manometría es de utilidad en disfagia de tipo motor.
corresponde a: En la panendoscopia se observó una estenosis
Divertículo epifrénico esofágica en tercio distal no franqueable con el
Divertículo de Zenker endoscopio. ¿La etiología más probable en esta
Divertículo por pulsión paciente es?
Diverticulosis Estenosis maligna
El divertículo en tercio superior de esófago es el de Zenker y se forma por un Estenosis péptica
mecanismo de pulsión. El epifrénico se localiza en tercio distal del esófago.
Esófago de Barrett
¿Cuál es el sitio típico de aparición de este divertículo?
Membrana esofágica
Cinco centímetros por debajo del esfínter esofágico Pacientes con enfermedad mixta del tejido conectivo cursan con trastornos
superior de la motilidad esofágica y síntomas típicos de reflujo. Debe de tener una
Triángulo de Killian estenosis péptica por reflujo.

Cinco centímetros por arriba del esfínter esofágico El tratamiento de elección en esta paciente es:
inferior En este momento no amerita tratamiento hasta que los
Puede aparecer en cualquier sitio del esófago síntomas empeoren
¿Cuál es el tratamiento en este caso? Valoración por cirugía
Tratamiento farmacológico Dilatación endoscópica
Diverticulectomía Miotomía
Estenosis no franqueable con síntomas amerita tratamiento con dilatación
Dilatación endoscópica para mejorar la disfagia
Pantoprazol Se realizó dilatación por endoscopía y se documentó
El tratamiento de elección en divertículo de Zenker es resección del
divertículo.
hernia hiatal de 4 cm. El tratamiento a largo plazo de
esta paciente será:
CASO CLÍNICO 6 Optimizar tratamiento con IBP
Mujer de 36 años de edad con enfermedad mixta del Enviar a cirugía para funduplicatura
tejido conectivo. Toma omeprazol de forma regular Dieta en tercios
por pirosis y regurgitación. Acude por disfagia de 4 Bajar cabecera
Por la enfermedad del tejido conectivo es probable que tenga problemas CASO CLÍNICO 2
esofágicos motores por lo que no sería una buena candidata para
funduplicatura. Su tratamiento es con IBP, dieta en quintos, elevar cabecera,
Hombre de 55 años de edad con consumo frecuente de
etc. alcohol desde los 20 años. Hace 10 meses acudió a
valoración de especialista porque presentaba diarrea
Dispepsia/ Úlcera péptica/ Gastritis/ Diarrea crónica y pérdida de peso. Los estudios de extensión
crónica demostraron insuficiencia pancreática por pancreatitis
CASO CLÍNICO 1 crónica.
Hombre de 28 años de edad tiene cuadros de diarrea con ¿Cuál sería el mecanismo fisiopatológico de esta
la ingesta de lácteos. diarrea?
¿A qué tipo de diarrea corresponde? Disminución en la superficie de absorción
Inflamatoria Mala digestión intraluminal
Secretora Inactivación de enzimas
Osmótica Hipomotilidad intestinal
La insuficiencia pancreática produce diarrea con datos de malabsorción por
Motora una mal digestión secundaria a una deficiencia de enzimas pancreáticas.
De acuerdo con lo anterior, ¿cuál es el mecanismo ¿Cómo esperaría encontrar los carotenos séricos en
fisiopatológico? este tipo de diarrea?
Daño e inflamación de la mucosa de colon Bajos
Secreción de agua y electrolitos Elevados
Sobrecrecimiento bacteriano Normales
Presencia de solutos no absorbibles Elevados 2 veces de lo normal
Principalmente es secundaria a presencia a solutos no absorbibles en el Carotenos bajos indican malabsorción. Pero no distingue entre las causas que
tracto gastrointestinal: la presencia de sustancias altamente osmóticas afecta la pared intestinal de las de origen pancreático.
permiten la salida de agua hacia el lumen disminuyendo la osmolaridad Si se le hiciera una prueba de D-xylosa, ¿qué resultado
intraluminal y esto disminuye los iones inorgánicos.
La etiología es: esperaría encontrar?
Tránsito intestinal acelerado D-xylosa <5 gr
Deficiencia enzimática D-xylosa <3 gr
Menor superficie de absorción D-xylosa >5 gr
Daño intestinal D-xylosa >4 gr
D xilosa distingue entre las causas que afectan la pared intestinal de las de
La intolerancia a los lácteos produce diarrea osmótica por solutos no
origen pancreático. Si es menor a 5 gramos indica afección de la pared
absorbibles por deficiencia de la enzima lactasa.
intestinal. Si es mayor a 5 sugiere mala digestión.
Es una característica de este tipo de diarrea: El tratamiento de elección en este paciente es:
Cede con el ayuno Loperamida
No cede con el ayuno Enzimas pancreáticas
Desequilibrio hidroelectrolítico Subsalicilato de bismuto
Evacuaciones con moco y sangre Colestiramina
Típicamente cede con ayuno lo que la distingue de la secretora que no cede.
Tratamiento es suplementar al paciente con enzimas pancreáticas en cada
¿Cuál de los siguientes estudios puede ayudar a alimento.
distinguir una diarrea osmótica de una secretora? ¿Qué porcentaje de pérdida de función pancreática se
Leucos en heces requiere para que un paciente tenga insuficiencia
K+ en heces pancreática exocrina?
Brecha aniónica 30%
A y B son correctas 60%
La brecha aniónica es de utilidad para diferenciarlas, siendo el punto de corte 80%
de 125.
90%
CASO CLÍNICO 3 2 meses, recibió tratamiento de erradicación. Se le tomaron
Mujer de 67 años, tiene hipertensión arterial sistémica biopsias gástricas que reportaron metaplasia intestinal
bajo tratamiento con amlodipino. Desde hace 6 meses incompleta en cuerpo y antro sin displasia. Actualmente se
presenta dolor abdominal epigástrico, plenitud encuentra asintomático.
¿Cuál de las siguientes es una indicación de erradicación de
temprana, edema de miembros inferiores. Sus
Helicobacter pylori?
laboratorios: Hb 13 g/dL, Cr 1 mg/dL, albúmina 2 g/dL, Antecedente de úlcera péptica
EGO sin proteinuria. Se le realizó una panendoscopia Metaplasia intestinal incompleta
en la que se observó engrosamiento de pliegues Linfoma gástrico
gástricos. Todas las anteriores
¿Cuál es el diagnóstico más probable? Usted quiere saber si se logró la erradicación de H.
Linfoma gástrico pylori. ¿Qué estudio solicitaría?
Síndrome de Zollinger-Ellison Serología para Helicobacter pylori
Gastroenteritis eosinofílica Prueba de aliento
Enfermedad de Ménétrier No es necesario corroborar erradicación ya que la tasa
El engrosamiento de pliegues gástricos aunado a la hipoalbuminemia de de respuesta es >90% con terapia triple
origen no renal sugiere una enteropatía perdedora de proteínas.
Con respecto a la hipoalbuminemia de esta paciente, Biopsia
La tasa de respuesta de erradicación de H. pylori es variable de acuerdo con el esquema
es cierto que: elegido y las resistencias antimicrobianas de cada población.
Es por pérdidas renales ¿Cuál sería el riesgo de metaplasia intestinal
Es secundaria a una enteropatía perdedora de proteínas incompleta a largo plazo?
No tiene relación con la sospecha clínica Úlcera péptica
Amerita recolección de orina de 24 hr Adenocarcinoma gástrico
Tiene EGO sin proteinuria y los hallazgos endoscópicos correlacionan con una Perforación gástrica
probable Enfermedad de Ménétrier.
No existe ningún riesgo
¿Cuál de los siguientes es un diagnóstico diferencial? Metaplasia intestinal incompleta con sulfocianina es el de mayor riesgo de cáncer
Cáncer gástrico gástrico.

Síndrome de Zollinger-Ellison ¿Qué tipo de metaplasia es la que tiene mayor riesgo


Gastropatía hipertrófica asociada a H. pylori de cáncer gástrico?
Todas las anteriores Metaplasia intestinal completa
Todas las anteriores dan engrosamiento de pliegues gástricos similar al Metaplasia intestinal incompleta
Ménétrier. Las dos presentan el mismo riesgo
El diagnóstico definitivo de esta patología es por: La metaplasia con sialomucinas +
Hallazgos endoscópicos En el seguimiento de este paciente será necesario:
Biopsia de pared gástrica Amerita seguimiento si se corrobora erradicación de H.
Por la hipoalbuminemia pylori
Diagnóstico de exclusión El pepsinógeno sirve como marcador tumoral
Tratamiento de esta patología es: Realizar panendoscopia de vigilancia
Gastrectomía No hacer nada
Vigilancia endoscópica Tiene metaplasia intestinal incompleta y extensa (afecta dos áreas del
Inhibidor de la bomba de protones estómago) por lo que amerita vigilancia por riesgo incrementado de
adenocarcinoma.
No amerita tratamiento
Tratamiento con gastrectomía por riesgo asociado de cáncer gástrico. Sin
embargo se puede optar por vigilancia endoscópica regular. CASO CLÍNICO 5
Hombre de 56 años de edad acude por melena de 3
CASO CLÍNICO 4 días de evolución. Se encuentra con TA 110/70mmHg,
Paciente masculino de 34 años, con antecedente de FC 110 lpm, FR 18 rpm, sin datos relevantes a la
úlcera péptica asociada a H. pylori documentada hace exploración física.
¿Cuál es el diagnóstico de este paciente? La gastrina es producida por:
Sangrado de origen desconocido Las glándulas oxínticas en estómago.
Sangrado de origen oculto Células parietales
Sangrado de tubo digestivo alto Células G
Sangrado de tubo digestivo bajo Células principales
La melena sugiere sangrado de tubo digestivo alto. No es de origen oscuro ni La gastrina se produce en el antro por células G. Las glándulas oxínticas
desconocido porque tiene sangrado manifiesto que no ha sido abordado. El contienen células principales que secretan pepsinógeno y las parietales que
sangrado de tubo digestivo bajo se manifiesta con hematoquezia o secretan FI y HCl.
rectorragia. ¿Cuál de las siguientes enzimas ese encarga de
La etiología más probable es: hidrolizar almidones?
Úlcera esofágica Amilasa
Mallory-Weiss Carboxipeptidasas
Esofagitis Nucleasas
Úlcera péptica Tripsina
El estudio de elección en este caso es: La única que hidroliza almidones es la amilasa que es una enzima glucolítica.
Colonoscopia El resto son proteolíticas.

Panendoscopia La distensión gástrica condiciona relajación del esfínter


Angiografía ileocecal. ¿Cómo se le llama a este reflejo?
Tomografía de abdomen Reflejo enterogástrico
El agente causal más probable es: Reflejo gastrocólico
Cirrosis hepática Reflejo gastroileal
Infección por H. pylori Reflejo entérico
Consumo frecuente de AINES Una mujer de 54 años de edad refiere deseo de
Gastritis biliar evacuar 30 min después del desayuno. ¿Cómo se le
H. pylori es la principal causa de ulcera péptica y en segundo lugar están los llama al siguiente reflejo?
AINES. Reflejo enterogástrico
En la panendoscopia se observó una úlcera en antro Reflejo gastrocólico
recubierta de fibrina. ¿Qué tratamiento requiere? Reflejo intestino-intestinal
Tratamiento hemostático por vía endoscópica Reflejo recto esfintérico
Sucralfato
Ranitidina Malabsorción intestinal / síndrome intestino irritable
Erradicación de H. pylori / Estreñimiento crónico / Isquemia mesentérica
Por ser Forrest III no amerita tratamiento endoscópico. Suponiendo que la
causa sea H. pylori habría que solicitar prueba de aliento y si sale positiva dar CASO CLÍNICO 1
tratamiento de erradicación, IBP durante 8 semanas. Hombre de 60 años inició hace 3 meses con diarrea
caracterizada por 5 evacuaciones diarias con
Preguntas independientes esteatorrea. Refiere que la diarrea mejora con el ayuno
Hormona que estimula principalmente la secreción de y empeora con los alimentos. Además de la diarrea, ha
enzimas pancreáticas por las células acinares: tenido pérdida de peso de 3 kg desde el inicio de su
Gastrina padecimiento actual. Tiene historia de hemicolectomía
Secretina derecha con anastomosis primaria a los 55 años por
Colecistocinina cáncer de colon.
Somatostatina ¿Cómo clasificaría clínicamente la diarrea del
Colecistocinina es el principal estimula para secreción de enzimas pancreáticas y la
secretina para la secreción de líquido rico en cloro y bicarbonato.
paciente?
Secretora
Inflamatoria
Malabsortiva
Osmótica
Se acompaña de esteatorrea, pérdida de peso y mejora con el ayuno. Si bien Carencia en la dieta de B12 e ingesta de suplementos de
la diarrea malabsortiva tiene un componente osmótico. En este caso el
mecanismo fisiopatológico primario es malabsorción. folato
El mecanismo de las deficiencias vitamínicas en este caso es consumo de
¿Cuál de las siguientes pruebas diagnósticas es de vitamina B12 por microbiota del intestino delgado y producción de ácido
utilidad para diferenciar malabsorción originada en la fólico por las mismas.
pared intestinal vs malabsorción intraluminal?
Prueba de aliento con lactulosa CASO CLÍNICO 2
Elastasa pancreática fecal Mujer de 45 años de edad con antecedente de
D-Xilosa estreñimiento crónico caracterizado por heces duras y
Secretina en jugo pancreático en escíbalos (Bristol 1-2), acompañadas de pujo y
La prueba inicial para diferenciar si la malabsorción tiene su origen en la tenesmo. Evacua diariamente y niega dolor
pared intestinal o el lumen intestinal es la D-Xilosa. La D-Xilosa es un
carbohidrato que se absorbe directamente al torrente sanguíneo en abdominal. Refiere que tiene dicho padecimiento
pacientes con pared intestinal intacta y se miden sus niveles en orina. desde los 18 años de edad y utiliza fibra con respuesta
Se realizó prueba de D-Xilosa y fue reportada por debajo del parcial. Es sana y no tiene otros antecedentes de
valor de referencia. Asimismo se midieron los niveles de importancia. Sus exámenes de laboratorio son
vitamina B12 que fueron reportados bajos y niveles de ácido normales.
fólico que fueron reportados por arriba del límite de
¿Qué prueba descartarías en el abordaje inicial de un
referencia. Se interrogó al paciente y no toma ningún
suplemento vitamínico. De acuerdo al cuadro clínico y
paciente con constipación?
antecedentes del paciente, ¿cuál es el mecanismo Manometría anorrectal
fisiopatológico más probable de la diarrea? Pruebas de función tiroidea
Pancreatitis crónica obstructiva Calcio sérico
Tumor en la cabeza del páncreas que obstruye el Tacto rectal
La manometría anorrectal no es un estudio en el abordaje inicial de la
conducto pancreático constipación. Se realiza después de haber descartado razonablemente causas
Sobrepoblación bacteriana secundarias ya sea clínicamente o con exámenes auxiliares.
Enfermedad celíaca Se le administró prueba terapéutica con Psyllium
El paciente tiene historia de resección de colon derecho con pérdida de la Plantago con respuesta parcial. ¿Cuál de los siguientes
válvula ileocecal. Esto ocasiona reflujo del contenido colónico al íleon
terminal y por lo tanto sobrepoblación bacteriana. Por otro lado las tratamientos es el más apropiado a continuación?
alteraciones en los niveles vitamínicos son sugerentes de sobrepoblación Enemas de lactosa
bacteriana. Biorretroalimentación
De acuerdo a su respuesta anterior, ¿cuál sería el Polietilenglicol
mejor tratamiento a ofrecer en este caso? Ningún tratamiento hasta no realizar colonoscopia
Cirugía de Whipple El tratamiento inicial de la constipación consiste en fibra soluble y laxantes
Dieta libre de gluten osmóticos. El polietilenglicol es un laxante osmótico. La lactosa también es
un laxante osmótico sin embargo la incomodidad de administrar enemas
Ciclos de antibióticos supera al beneficio.
Pancreatografía con colocación de endoprótesis Se inició el tratamiento recomendado por usted sin mejoría
El mejor tratamiento para sobrepoblación bacteriana es antibióticos cíclicos.
adicional. La paciente refiere que adicionalmente tiene
Son de elección rifaximina, ciprofloxacino, amoxicilina con clavulanato,
tetraciclinas, metronidazol. De preferencia deben alternarse los antibióticos episodios de dolor durante la defecación y a veces requiere
para disminuir el riesgo de resistencia bacteriana. introducir su dedo para extraer materia fecal residual y pujo
¿Cuál es el mecanismo fisiopatológico de las excesivo con tenesmo. ¿Cuál de las siguientes pruebas
alteraciones en los niveles vitamínicos? diagnósticas se encuentra indicada a continuación?
Anticuerpos anti células parietales/ingesta oculta de Colonoscopia
suplementos de folato Enema baritado
Resección de íleon distal Manometría anorrectal
Consumo bacteriano de B12/producción bacteriana de Tránsito colónico
La paciente tiene síntomas sugerentes de disinergia defecatoria u
ácido fólico obstrucción funcional. En estos casos el abordaje de elección es manometría
anorrectal.
¿Cuál es el mecanismo fisiopatológico más probable de Antiespasmódicos
los síntomas de la paciente? Antidepresivos tricíclicos
Obstrucción anorrectal por tumor El tratamiento del síndrome de intestino irritable es de acuerdo al síntoma.
El tratamiento inicial de elección para el dolor abdominal son los
Contracción paradójica del puborrectal antiespasmódicos.
Tránsito colónico lento con inercia colónica Pasados 4 meses, se agregó al cuadro del paciente,
Mal apego al tratamiento además del dolor abdominal, episodios de distensión
Uno de los mecanismos de disinergia defecatoria es contracción paradójica
del puborrectal.
abdominal y hábito defecatorio alternante con diarrea
¿Cuál es el tratamiento de elección para la condición (25%) / estreñimiento (30%). Se realizaron exámenes
de la paciente? microbiológicos de materia fecal sin encontrar
Biorretroalimentación alteraciones. No hay cambios en otros síntomas y los
Agregar procinético laboratorios generales son normales. ¿Cuál de las
Enemas periódicos siguientes pruebas diagnósticas considera apropiada
Resección anterior baja de tumor colónico para solicitar a continuación?
El tratamiento de elección de los trastornos funcionales defecatorios es la Colonoscopia
biorretroalimentación. Anticuerpos anti-transglutaminasa
Sangre oculta en heces
CASO CLÍNICO 3 Tránsito intestinal
Hombre de 32 años con dolor abdominal tipo cólico Una forma de manifestación de enfermedad celíaca es como simulador de
intermitente desde hace 2 años, asociado a episodios síndrome de intestino irritable variedad diarrea por lo tanto, en este caso
está indicado realizar tamizaje con AC anti TG IgA.
de constipación. El dolor se presenta 1 vez por semana
y mejora con las evacuaciones. Los períodos sin
CASO CLÍNICO 4
síntomas sus evacuaciones son normales, niega otros
Mujer de 80 años, obesa, diabética e hipertensa, con
síntomas y sus exámenes de laboratorio son normales.
historia de infarto agudo al miocardio hace 10 años
¿Cuál es el diagnóstico más probable en este caso?
tratado con angioplastía con colocación de stent
Constipación funcional
medicado. Inició hace 6 meses con dolor abdominal
Síndrome de intestino irritable
intermitente postprandial (1 hora después de comer).
Cáncer de recto
Dicho dolor es intenso y la paciente disminuyó su
Disinergia defecatoria
El paciente cumple con criterios de Roma para SII. Hay que recordar que es
ingesta por miedo al dolor. Durante este período ha
necesario el dolor abdominal para integrar el diagnóstico de SII. bajado 8 kg de peso.
De acuerdo a su diagnóstico, ¿cuál es una indicación de ¿Cuál es el mecanismo subyacente más probable de los
realizar colonoscopia en pacientes con este síntomas de la paciente?
padecimiento? Trombosis venosa mesentérica
Todos los pacientes con constipación Trombosis arterial mesentérica
Todos los pacientes con SII Placa ateromatosa arterial mesentérica
Edad mayor a 50 años Obstrucción intestinal intermitente
Tenesmo durante las evacuaciones Es un caso típico de angina intestinal o isquemia mesentérica crónica (Dolor
El síndrome de intestino irritable es un diagnóstico clínico y en ausencia de postprandial y sitofobia). La angina intestinal es secundaria a disminución
síntomas de alarma puede diagnosticarse razonablemente sin exámenes predecible del flujo sanguíneo por obstrucción fija (placa ateromatosa) en la
auxiliares. La excepción es en pacientes con diarrea (Pruebas de función circulación arterial mesentérica. En este caso la paciente adicionalmente
tiroidea y anticuerpos anti-transglutaminasa) y en pacientes con datos de tiene múltiples factores de riesgo de enfermedad cardiovascular que
alarma o mayores de 50 años (colonoscopia). usualmente se encuentran presentes en estos casos.

¿Cuál opción considera apropiada como primera Dos días después, la paciente acudió al servicio de urgencias
por dolor abdominal de 3 horas de evolución sin relación con
opción para el tratamiento del dolor abdominal en
los alimentos. La exploración física únicamente con ruidos
este caso? intestinales discretamente disminuidos en intensidad pero
Fibra soluble la paciente tiene un dolor desproporcionado a los hallazgos.
Fibra no soluble
¿Cuál es el mecanismo más probable en este caso del cuadro es la causa más probable del cambio en el patrón de los
agudo? síntomas de la paciente?
Embolismo arterial mesentérico Nueva trombosis arterial mesentérica
Trombosis arterial mesentérica Trombosis venosa mesentérica
Placa ateromatosa que incrementó su tamaño Isquemia colónica
Íleo por congestión capilar del intestino delgado Enfermedad diverticular
Si bien el mecanismo más frecuente de isquemia mesentérica aguda es La paciente tuvo un episodio de deshidratación con inestabilidad
embolismo arterial. En este caso existe un factor de riesgo claro para hemodinámica que muy probablemente condicionó hipoperfusión a las
trombosis in situ (presencia de placa ateromatosa). zonas penumbra del colon (Sudeck y Griffith). La isquemia colónica es el
¿Cuál es el siguiente estudio recomendado en este síndrome más frecuente de isquemia mesentérica.

caso?
Angiotomografía de abdomen CASO CLÍNICO 5
Resonancia magnética de abdomen Hombre de 35 años de edad. Inició hace 8 meses con
Angiografía mesentérica urgente dolor abdominal intermitente, tipo cólico,
Angiografía mesentérica electiva acompañado de evacuaciones duras con pujo y
El estudio inicial en un paciente con isquemia mesentérica sin datos de tenesmo. El dolor mejora con la defecación. Niega
irritación peritoneal y con menos de 12 horas de evolución de los síntomas otros síntomas y durante los períodos libres de dolor
es angiotomografía de abdomen. Es útil para planear el tratamiento ulterior.
En caso de encontrar datos de perforación, neumatosis o gas portal se abdominal su hábito intestinal es normal.
requeriría tratamiento quirúrgico. En caso de no encontrar datos sugerentes ¿Cuál de los siguientes es el diagnóstico diferencial
de perforación o de intestino no viable, el siguiente paso es angiografía. más apropiado en este caso?
El estudio mencionado, mostró obstrucción total al flujo
Constipación crónica funcional con disinergia
sanguíneo cerca del ostium de la arteria mesentérica
superior. No hay aire libre subdiafragmático ni datos de
defecatoria
neumatosis intestinal/portal. Las condiciones de la paciente Constipación crónica funcional de tránsito colónico
no han cambiado desde su llegada (hace 1 hora) ¿Cuál es el lento
siguiente paso en el tratamiento de la paciente? Síndrome de intestino irritable – constipación
Anticoagulación y vigilancia Constipación crónica funcional de tránsito colónico
Anticoagulación y angiografía normal
Laparotomía exploratoria La presencia de dolor abdominal que se relaciona con la defecación y con el
cambio en el hábito intestinal es altamente sugerente de síndrome de
Antiagregantes y vigilancia intestino irritable. Este paciente no tiene datos de alarma que obliguen a
El estudio auxiliar (TAC), mostró obstrucción probablemente secundaria a realizar un abordaje paraclínico extenso.
trombosis in situ de placa de ateroma. De acuerdo a los hallazgos, ausencia ¿Cuál de las siguientes características clínicas es
de irritación peritoneal y datos de perforación se sugiere angiografía
diagnóstica/terapéutica (trombectomía, angioplastía e infusión de indispensable para integrar el diagnóstico previo?
papaverina). Pujo
El tratamiento recomendado fue exitoso y la paciente fue Tenesmo
egresada a la semana de su ingreso y se ha mantenido Dolor abdominal
asintomática en el seguimiento. Un año después, acude
Constipación
nuevamente a urgencias por diarrea acuosa abundante (10 La presencia de dolor abdominal es fundamental para poder integrar el
evacuaciones diarias) después de viaje. En la exploración diagnóstico de síndrome de intestino irritable.
física se evidencia ortostatismo y sequedad de mucosas. Los 30 años después, inició con constipación de reciente
exámenes de laboratorio destacan por Na 128 mEq/L, Cr 2.5 aparición caracterizada por evacuaciones acintadas y
(previa de 1.2), se realizaron coprocultivos y disminuidas en frecuencia con pujo y tenesmo, se
coproparasitoscópicos que fueron negativos. Se realizó
mantuvo estable con tratamiento sintomático durante
hidratación intravenosa con mejoría, sin embargo 2 días
después, inició con dolor abdominal tipo cólico súbito en
esos años. Al interrogatorio refiere que el cuadro
mesogastrio, acompañado de una evacuación apareció desde hace 2 meses y ha empeorado. No
hematoquézica en moderada cantidad. Los síntomas siempre se acompaña de dolor abdominal. ¿Cuál de los
duraron 1 día y se autolimitaron con mejoría ulterior. ¿Cuál siguientes diagnósticos es el menos probable dentro
del diagnóstico diferencial en este caso?
Constipación funcional ¿Cuál es la causa más probable de los síntomas
Carcinoma colorrectal gastrointestinales en este paciente?
Hipercalcemia Isquemia mesentérica aguda arterial por embolismo
Hipotiroidismo Isquemia mesentérica crónica por placa ateromatosa
La forma de presentación sugiere una constipación secundaria. Todas las Isquemia / reperfusión colónica
presentadas son causas secundarias, excepto constipación funcional.
Isquemia mesentérica por trombosis venosa
Este examen complementario se puede evitar Se trata de un cuadro típico de colitis isquémica. La causa más frecuente de
realizarse de manera inicial: isquemia intestinal es la isquemia colónica y el 95% de los casos se debe a
Colonoscopia cuadros no oclusivos de la circulación. En este caso existen dos factores claros
que pudieron contribuir (choque cardiogénico y procedimiento endovascular
Pruebas de función tiroidea arterial).
Electrolitos séricos Este hallazgo no corresponde a alguna de las pruebas
Manometría anorrectal diagnósticas en este caso:
Antes de realizar abordaje para constipación funcional y trastornos
defecatorios en un paciente >50 años con cambio reciente en el hábito
Obstrucción total del flujo de la arteria mesentérica
intestinal, se sugiere descartar causas secundarias. superior
¿En cuál de los siguientes escenarios clínicos solicitaría Mucosa edematosa, frágil con úlceras longitudinales y
una colonoscopia desde el inicio del abordaje? áreas de hemorragia.
Mujer de 40 años con constipación de 20 años de Signo de la huella digital en TAC
evolución Engrosamiento de colon izquierdo
Mujer de 45 años con pujo y tenesmo de 15 años de La localización más frecuente de la isquemia colónica es el colon izquierdo. El
estudio inicial de elección es la TAC y se sugiere realizar colonoscopia
evolución temprana para corroborar el diagnóstico. Recordando esto y que la causa
Mujer de 30 años con constipación acompañada de más frecuente de isquemia colónica es no oclusiva, es poco probable en un
dolor abdominal intermitente desde hace 5 años que paciente de estas características y con evolución favorable tenga datos por
imagen de trombosis arterial mesentérica.
mejora al evacuar Esta área del intestino puede tener menos
Mujer de 40 años con constipación de reciente susceptibilidad a la isquemia:
aparición acompañada de síndrome anémico sin Unión rectosigmoidea (Punto de Sudeck)
sangrado manifiesto Ángulo esplénico (punto de Griffith)
La presencia de datos de alarma (anemia, pérdida de peso, cambios recientes
en el hábito intestinal, antecedentes familiares de cáncer de colon, sangrado) Recto
son sugerentes de causas secundarias de constipación y requieren mayor Yeyuno
abordaje. El recto al tener doble circulación (AMS y AMI) es poco probable que sufra
eventos de isquemia.
CASO CLÍNICO 6 Puedes diferir la cirugía, si se presenta este signo:
Hombre de 50 años, hospitalizado por Infarto agudo al Neumatosis intestinal
miocardio. Como manifestación inicial tuvo choque Neumatosis portal
cardiogénico. Recibió terapia de reperfusión mediante Necrosis colónica en colonoscopia
cateterismo con colocación de stent medicado en Hematoquezia moderada
arteria descendente anterior. Ha evolucionado Sólo la hematoquezia masiva es una indicación de cirugía
favorablemente, egresó de la unidad coronaria a piso ¿Cuál es la forma más común de comportamiento de
y se encuentra esperando su alta hospitalaria, sin esta entidad?
embargo inició hace 2 horas con dolor abdominal tipo Evolución favorable y resolución
cólico seguido por 2 evacuaciones hematoquézicas. A Deterioro y perforación intestinal
la exploración: signos vitales normales, no hay datos Mejoría inicial con deterioro ulterior
de irritación peritoneal. El paciente se negó a recibir Progresión rápida con sepsis y choque séptico
abordaje diagnóstico y continuó únicamente con el La mayoría de los casos de colitis isquémica tienen un curso benigno y
autolimitado. Incluso muchos casos no se diagnostican debido a que el
tratamiento ya establecido a base de anticoagulación paciente no acude a consulta. Los casos con peor pronóstico son los casos
y antiagregantes por el síndrome coronario agudo. Su con isquemia del colon derecho, SRIS, leucocitosis o neumatosis en los
estudios de imagen.
evolución fue favorable y fue egresado a las 48 horas.
Oclusión intestinal/Enfermedad Inflamatoria A menos de que haya datos de irritación peritoneal o hallazgos en la imagen
que sugieran perforación, el manejo inicial de una oclusión intestinal es
Intestinal/Hepatopatía por alcohol conservador.
CASO CLÍNICO 1 La paciente mejoró después de 24 horas con el manejo
Mujer previamente sana de 70 años de edad, inició indicado. ¿Cuál es el pronóstico en este caso?
hace 5 horas con dolor abdominal tipo cólico intenso. La tasa de recurrencia es nula
Posteriormente se agregó al cuadro vómito abundante Se espera recurrencia de aproximadamente 20%
de contenido gastrobiliar, distensión abdominal e La tasa de recurrencia es de aproximadamente 60%
imposibilidad para evacuar. A la exploración destaca La tasa de recurrencia es de 5%
la presencia de ruidos metálicos, sin datos de irritación La mayoría de los estudios de recurrencia de oclusión intestinal refieren una
tasa de aproximadamente 20% en los siguientes 5 a 10 años del evento.
peritoneal. Tiene antecedentes quirúrgicos de
colecistectomía, apendicectomía e histerectomía
Además de los exámenes generales, ¿cuál es el primer CASO CLÍNICO 2
estudio a solicitar en este caso? Mujer de 29 años de edad con bulimia sin
Radiografía de abdomen antecedentes de importancia ni otras comorbilidades.
Tomografía de abdomen Tiene historia de uso de múltiples métodos
Ultrasonido abdominal “purgativos”. Inició hace 6 horas con vómito de
Serie esofagogastroduodenal contenido gastrobiliar, distensión abdominal e
En un cuadro típico de oclusión intestinal el primer estudio a solicitar es una imposibilidad para evacuar. En la exploración física se
radiografía de abdomen. En caso de tener dudas respecto al diagnóstico o encuentra ausencia de ruidos intestinales. No hay
bien clarificar y buscar causas, el siguiente estudio es una tomografía de
abdomen.
datos de irritación peritoneal.
¿Cuál de los siguientes hallazgos es el más probable en ¿Cuál es el mecanismo fisiopatológico más probable en
el estudio solicitado? este caso?
Niveles hidroaéreos sin gas en el colon Mecánico
Niveles hidroaéreos con gas en el colon Metabólico
Líquido libre intraabdominal Psicógeno
Aire libre subdiafragmático Isquémico
Lo más probable es que se trate de una oclusión intestinal mecánica. El Es un cuadro típico de obstrucción intestinal motora (íleo paralítico). Después
hallazgo típico por radiografía es presencia de niveles hidroaéreos sin gas en del íleo postquirúrgico donde el antecedente de cirugía es evidente, las
el colon. causas metabólicas son las más frecuentes. La isquemia también puede
ocasionar íleo sin embargo debido a los antecedentes de la paciente y el
¿Cuál es la etiología más probable del cuadro de la cuadro clínico, la causa más probable es metabólica (hipocalemia).
paciente? Usted solicita radiografía de abdomen, ¿cuál de los
Hipocalemia siguientes hallazgos espera encontrar con mayor
Isquemia mesentérica probabilidad?
Adherencias Dilatación difusa de asas con gas colónico
Medicamentos Dilatación difusa de asas sin gas colónico
La paciente tiene historia de tres cirugías abdominales. Además, la causa más
frecuente de obstrucción intestinal mecánica es adherencias por cirugías
Dilatación difusa de asas con zona de transición clara
abdominales previas. Aire libre subdiafragmático
¿Cuál no es una opción terapéutica apropiada en este El patrón radiológico típico de la obstrucción intestinal motora (íleo
paralítico) es dilatación difusa de asas con niveles hidroaéreos con presencia
momento? de gas en el colon. La ausencia de gas y la presencia de zona de transición son
Laparotomía exploradora sugerentes de oclusión mecánica.
Hidratación Además del cuadro clínico principal, la paciente tiene
Colocación de sonda nasogástrica dolor intenso del tobillo por esguince a la hora de
Analgesia bajarse del coche hacia el hospital. ¿Cuál de los
siguientes tratamientos estaría contraindicado en este La causa más frecuente de síndrome disentérico son las infecciones. En el
abordaje inicial en un paciente joven con síndrome disentérico sin otros
caso? datos de alarma no está indicado de forma inicial la realización de
Colocación de sonda nasogástrica colonoscopia. Dicho estudio debe realizarse una vez que no se encuentran
causas infecciosas como abordaje de sangrado de tubo digestivo bajo.
Hidratación y reposición de electrolitos
En el abordaje no se encontraron causas infecciosas. El
Analgesia con AINES
paciente decidió irse de alta voluntaria y se automedicó con
Analgesia con opiáceos
antibióticos. Regresó 6 meses después por persistencia de
En caso de requerir analgesia por cualquier motivo, el uso de opiáceos no es
deseable debido a que contribuirían a perpetuar la oclusión intestinal los síntomas. Usted decide realizar colonoscopia. ¿Qué
motora. hallazgos esperaría usted encontrar?
Después de tranquilizar a la paciente, se procede con un Eritema y pérdida del patrón vascular que respetan el
interrogatorio minucioso, ¿cuáles de los siguientes métodos recto
purgativos utilizados por la paciente, es más probable que Úlceras en botón de camisa
haya contribuido al cuadro? Eritema y pérdida del patrón vascular que se extiende
Vómito y uso de diuréticos de distal a proximal
Ejercicio anaeróbico y uso de laxantes estimulantes Úlceras serpiginosas alternando con áreas de mucosa
Atracón reciente y consumo de fentermina sana
Uso de laxantes estimulantes y atracones El cuadro clínico es altamente sugestivo de Colitis ulcerativa crónica
El vómito y el uso de diuréticos son factores que contribuyen al desarrollo de inespecífica (CUCI: hematoquezia con pujo y tenesmo). Dicha enfermedad se
hipocalemia que es una etiología clara de íleo paralítico. caracteriza por afección universal del recto, continua, circunferencial y de
¿Cuál de las siguientes alteraciones de laboratorio, distal a proximal..
espera encontrar con mayor probabilidad en este La biopsia reportó colitis crónica con plasmocitosis
caso? basal con ramificación de criptas. ¿Cuál de las
Acidosis metabólica e hipercalemia siguientes no es una característica típica de esta
Alcalosis metabólica e hipocalemia enfermedad?
Alcalosis respiratoria e hipocalemia Afección únicamente del colon
Alcalosis metabólica e hipercalcemia Ileítis retrógrada en algunos casos
Debido a las causas subyacentes en este caso del íleo paralítico, lo más Enteritis segmentaria en cualquier sitio del tubo
frecuente en un paciente con vómito y/o consumo de diuréticos, es la
digestivo
presencia de alcalosis metabólica hipocalémica.
Inflamación circunferencial continúa en recto
La CUCI se caracteriza por afección circunferencial continua y simétrica del
CASO CLÍNICO 3 recto que se extiende de distal a proximal. Dicha entidad afecta únicamente
Hombre de 25 años de edad sin antecedentes de el colon y en aproximadamente 20% de los casos de colitis extensa
(pancolitis) puede observarse ileítis retrógrada. La enteritis segmentaria es
importancia. Inició hace 1 semana con evacuaciones una característica de la enfermedad de Crohn.
sanguinolentas intermitentes en escasa cantidad, ¿Cuál de los siguientes factores es poco probable que
acompañadas de pujo y tenesmo por lo que acude a haya contribuido al cuadro?
consulta. A la exploración física, paciente se encuentra Suspensión de tabaquismo de larga evolución
con signos vitales normales, no tiene dolor abdominal Tabaquismo de larga evolución
a la palpación y no hay alteraciones al tacto rectal, Uso de AINES
aunque en el guante se observan restos hemáticos. Raza caucásica
¿Cuál de las siguientes no es una conducta apropiada El tabaquismo se considera un factor protector para el desarrollo de CUCI.
en este momento? Sólo 13% de los pacientes con CUCI son fumadores. La mayoría son no
fumadores o fumadores previos. Existe un gradiente de norte a sur para
Realizar coproparasitoscópico y coprocultivo enfermedad inflamatoria intestinal con mayor prevalencia en los países con
Toxina de C. difficile razas caucásicas.
Realizar colonoscopia
Medición de reactantes de fase aguda
Este fármaco es inapropiado en el tratamiento inicial ¿Cuál de las siguientes manifestaciones
de esta enfermedad: extraintestinales no se encuentra asociada a esta
Prednisona entidad?
Azatioprina Uveítis
Mesalazina Sacroileítis
Sulfasalazina Eritema nodoso
Las tiopurinas (azatioprina y mercaptopurina) no se consideran fármacos Dermatitis herpetiforme
para inducción de la remisión en CUCI debido a que el tiempo de inicio de La dermatitis herpetiforme se encuentra asociada a enfermedad celíaca no a
acción es muy prolongado (aprox. 6 semanas). Los esteroides y los Enfermedad de Crohn.
aminosalicilatos son medicamentos pilares para la inducción a la remisión en
este grupo de pacientes. ¿Cuál de los siguientes es el sitio afectado más
frecuente en esta enfermedad?
CASO CLÍNICO 4 Recto
Mujer de 24 años de edad. Inició hace 2 años con Íleon distal y colon derecho
episodios de dolor abdominal localizado en fosa ilíaca Estómago y duodeno
derecha. Ha acudido a urgencias en dos ocasiones ante Yeyuno
La localización más frecuente de la enfermedad de Crohn es la ileocolónica.
la posibilidad de apendicitis sin embargo dicho
¿Cuál medicamento descartarías para el manejo de
diagnóstico se ha descartado. En uno de los estudios
esta enfermedad?
de imagen se observó engrosamiento en los últimos 30
Mofetil micofenolato
cm del íleon terminal. En esta ocasión acudió al
Prednisona
presentar 2 evacuaciones hematoquézicas por lo que
Budesonida
se realizó colonoscopia encontrando úlceras lineales
Infliximab
profundas en íleon terminal y en colon derecho con El mofetil micofenolato no es un medicamento utilizado en el manejo de
áreas de mucosa sana circundante. pacientes con esta enfermedad.
¿Cuál de las siguientes aseveraciones no corresponde
a esta enfermedad? CASO CLÍNICO 5
Afección transmural Hombre de 55 años acude al servicio de urgencias debido
Afección exclusiva del colon e íleon distal a ictericia, fiebre y dolor en hipocondrio derecho. A la
Afección de cualquier sitio del tubo digestivo exploración se encuentra taquicárdico. La exploración de
Formación de fístulas enterocutáneas abdomen muestra hepatomegalia y dolor a la palpación
La enfermedad de Crohn puede afectar cualquier sitio del tubo digestivo y su del hipocondrio derecho. Tiene antecedente de uso de
afección es transmural. Existen los fenotipos no perforante no estenosante, valproato por crisis convulsivas y niega ingesta de
estenosante y penetrante. Este último se asocia a la formación de fístulas.
alcohol. En los exámenes de laboratorio, destaca
¿Cuál de los siguientes hallazgos histopatológicos es
hiperbilirrubinemia de 15 mg/dL, BD 7 mg/dL, BI 8 mg/dL,
característico de esta enfermedad?
FA 130 U/L, GGT 400 U/L. No hay reactivo para medición
Plasmocitosis basal con ramificación de criptas de aminotransferasas. BH con Hb de 11 g/dL con VGM de
Granulomas caseificantes 110, leucocitos de 25000 / µL, INR 2.6. Creatinina 0.8
Granulomas no caseificantes mg/dL. Se realizó ultrasonido de hígado y vías biliares
Células inflamatorias con núcleos gigantes y cuerpos de observando hígado brillante sin dilatación de la vía biliar
inclusión y la vesícula biliar sin litos.
La presencia de granulomas no caseificantes es altamente sugestiva de
enfermedad de Crohn, lamentablemente su rendimiento diagnóstico es bajo
ya que solo se logran encontrar en 5% de las biopsias mucosas y en 30% de
los especímenes de resección intestinal.
¿Qué diagnóstico descartarías por no corresponder ¿Cuál de las siguientes terapéuticas te parece la menos
con el cuadro? viable en este caso?
Hepatitis viral Prednisona
Lesión hepática inducida por fármacos Abstinencia
Hepatitis alcohólica Pentoxifilina
Adenocarcinoma de páncreas Trasplante hepático
El cuadro clínico es sugestivo de inflamación del parénquima hepático. En Para considerar a un paciente candidato a trasplante hepático es necesario
este momento dentro del diagnóstico diferencial debemos tomar en cuenta contar con abstinencia de por lo menos 6 meses. Los esteroides son el
cualquier causa de hepatitis aguda (virus, toxinas, alcohol). No hay dilatación tratamiento de primera línea en hepatitis alcohólica y la pentoxifilina puede
de la vía biliar y el patrón de las PFH no sugiere en absoluto causas utilizarse en pacientes con falla renal, infecciones, sangrado o donde el uso
obstructivas (cáncer de páncreas). de esteroide se encuentra contraindicado.
Se realizó biopsia hepática transyugular, observando
esteatosis macrovesicular con infiltrado neutrofílico. CASO CLÍNICO 6
Con esta información y el cuadro clínico, ¿cuál sería su Hombre de 50 años con historia de consumo de alcohol
primer diagnóstico diferencial? excesivo. Acude a urgencias por incremento en el
Lesión hepática por valproato perímetro abdominal de 1 semana de evolución
Esteatohepatitis no alcohólica acompañado de edema de miembros inferiores.
Esteatohepatitis alcohólica La presencia de este hallazgo descarta patología por
Colangitis ascendente consumo de alcohol excesivo:
Muchos pacientes niegan la ingesta de alcohol inicialmente. La Hipertrofia parotídea
esteatohepatitis alcohólica es difícil de distinguir histológicamente de la no
alcohólica sin embargo la presentación de la segunda, no es aguda con Anemia macrocítica
hiperbilirrubinemia, fiebre y leucocitosis. Además existen varias pistas que Anillo de Kayser-Fleischer
orientan a consumo de alcohol (elevación de la GGT, macrocitosis en la BH).
Elevación de la GGT
¿Cuál es la fisiopatología subyacente más probable en El anillo de Kayser-Fleischer se observa en la enfermedad de Wilson.
este caso? Al interrogatorio dirigido, el paciente refiere que
Toxicidad por acetaldehído y estrés oxidativo tomaba aproximadamente 300 mL de ron de 38GL
Esteatosis macrovesicular por síndrome metabólico diariamente durante varios años. ¿Cuál es el consumo
Hipersensibilidad al valproato diario aproximado en gramos de alcohol de este
Obstrucción del colédoco con infección bacteriana paciente?
secundaria 50 gramos
El principal mecanismo de toxicidad del alcohol es toxicidad por
20 gramos
acetaldehído. La producción de radicales libres mediante el sistema
microsomal produce daño y estrés oxidativo. 60 gramos
El laboratorio analizó la muestra inicial congelada para 90 gramos
Para calcular el consumo de alcohol en gramos se multiplica la cantidad de la
completar las pruebas de función hepática. ¿Cuál de
bebida en mL por el porcentaje de alcohol, en este caso (0.38) ya que se trata
los siguientes patrones de aminotransferasas es de una bebida de 38 grados GL. El resultado es el consumo de alcohol en mL.
probable encontrar con mayor probabilidad en este Posteriormente para convertir mL a gramos se multiplica por 0.78 (1 mL de
alcohol pesa 0.78 gramos). En este caso 300 mL x 0.38=114 mL de alcohol.
caso? 114 x 0.78=88.92 gramos.
ALT 500 U/L, AST 450 U/L ¿Cuál es la ingesta máxima diaria recomendada de
ALT 2500 U/L, AST 3000 U/L alcohol?
ALT 100 U/L, AST 220 U/L 60 gramos de alcohol en hombres
ALT 125 U/L, AST 100 U/L 30 gramos de alcohol en hombres
La hepatitis alcohólica de forma característica no eleva las aminotransferasas
de forma masiva. Usualmente son elevaciones de leves a moderadas y el 30 gramos de alcohol en mujeres
patrón característico es un predominio de la AST sobre la ALT (2:1). 20 gramos de alcohol en hombres
La ingesta máxima recomendada diariamente es de 20 gramos en mujeres y Destrucción biliar con lesión ductal florida
30 en hombres. A partir de 60 gramos el riesgo de esteatohepatitis alcohólica
aumenta considerablemente.
Ductopenia
¿Cuál es el tratamiento que se ha asociado a mejor Hepatitis lobulillar con polimorfonucleares
Hepatitis de interfase con células plasmáticas
pronóstico en estos casos? De acuerdo al cuadro clínico y los hallazgos, parece que se trata de una
Trasplante hepático temprano hepatitis autoinmune. El hallazgo histológico típico de esta entidad es
hepatitis de interfase. En algunos casos puede observarse lobulillar sobre
Abstinencia todo en etapas tempranas pero el infiltrado no es neutrofílico y usualmente
Uso de disulfiram se observa más en pacientes con falla hepática aguda.
Baclofeno ¿Cuál es la explicación de la elevación de las globulinas
La abstinencia es el pilar del tratamiento en cirrosis alcohólica. Los demás plasmáticas en esta paciente?
tratamientos se consideran auxiliares, algunos para lograr abstinencia, otros Translocación bacteriana
para tratar complicaciones.
Elevación monoclonal de la IgG
El paciente continuó bebiendo por 2 años más. Es
Elevación policlonal de la IgG
llevado por sus familiares a urgencias por Elevación policlonal de la IgM
encefalopatía. Actualmente lleva 1 semana sin tomar. La hepatitis autoinmune característicamente se asocia a hiperglobulinemia
Tiene ictericia, encefalopatía grado 3 con rueda IgG.

dentada y ascitis a tensión. Este paciente no es ¿Cuál es el tratamiento inicial de elección para esta
entidad?
candidato idóneo para:
Esteroides y azatioprina
Endoscopia
Azatioprina
USG hepático Mofetil micofenolato
Paracentesis Tacrolimus
Evaluación de trasplante hepático El tratamiento inicial de la hepatitis autoinmune puede ser con esteroide
como monoterapia en dosis altas (60 mg por día de PDN) o bien la
combinación de esteroide con azatioprina. Esta combinación ayuda a ahorrar
Hepatitis no infecciosa / Cirrosis/ Tumores de hígado dosis de esteroide y es la más utilizada en la actualidad. Los pacientes con
CASO CLÍNICO 1 cirrosis y colestasis tienen mayor riesgo de desarrollar toxicidad por
tiopurinas por lo que en estos pacientes se prefiere sólo esteroide.
Mujer de 23 años de edad previamente sana. Inició
¿Cuáles son las metas terapéuticas y la duración
hace 3 meses con fatiga y artralgias. Posteriormente
necesaria del tratamiento en esta paciente?
se agregaron al cuadro dolor en hipocondrio derecho
El tratamiento puede suspenderse al normalizar las
por lo que acude a consulta. Exámenes de laboratorio:
transaminasas y las globulinas después de 6 meses
ALT 300 U/L, AST 280 U/L, FA 120 U/L, BT 1.8 mg/dL,
El tratamiento puede suspenderse al normalizar las
BD 0.8 mg/dL, BI 1 mg/dL. Globulina de 6 g/dL.
transaminasas y globulinas después de 18 meses
¿Cuál de los siguientes auxiliares de diagnóstico
La suspensión del tratamiento requiere nueva biopsia
evitarías en la evaluación inicial de esta paciente?
para confirmar remisión histológica y su duración es de
Ultrasonido de hígado y vías biliares
2 años
Colangiorresonancia magnética
El tratamiento debe continuar de por vida en todos los
Serología para VHA
pacientes en dosis de mantenimiento al lograr
Serología para VHE
Se trata de una paciente con un cuadro clínico de hepatitis aguda. El patrón
normalización de transaminasas
de las PFH es hepatocelular y no hay datos de colestasis por lo que no está Para suspender el tratamiento se requiere una duración de 2 años y debe
justificado realizar colangiorresonancia. El abordaje inicial de la hepatitis confirmarse remisión histológica antes de suspenderlo.
aguda requiere descartar causas virales.
Los exámenes complementarios no mostraron CASO CLÍNICO 2
alteraciones. Se solicitaron autoanticuerpos encontrando Mujer de 44 años de edad con historia de
ANAs positivos (1:160), AML positivos (1:160). El hipotiroidismo en tratamiento actual con levotiroxina.
hepatólogo sugirió realización de biopsia hepática. ¿Cuál Inició hace 3 meses con fatiga y prurito. El prurito es
de los siguientes hallazgos histológicos espera usted incapacitante. Su esposo refiere que los últimos meses
encontrar?
ha notado un cambio discreto en la coloración de la Las lesiones amarillentas son xantomas por hipercolesterolemia. Los
pacientes con CBP tienen niveles elevados de colesterol y curiosamente no
piel de la paciente y la salida de manchas amarillentas tienen un riesgo cardiovascular elevado.
en los párpados. En los laboratorios destaca BT 1
mg/dL, FA 800 U/L, GGT 500 U/L, Albúmina 4 g/dL, CASO CLÍNICO 3
Globulinas 5 g/dL. Hombre de 240 años de edad previamente sano. Tiene
¿Cuál de los siguientes hallazgos en los exámenes como antecedente un hermano fallecido a los 18 años
complementarios, esperaría usted encontrar con de edad por “hepatitis aguda” y una hermana de 30
mayor probabilidad en esta paciente? años con trastorno del movimiento no diagnosticado y
Hiperglobulinemia a expensas de IgG cirrosis hepática. Inició hace 10 semanas con fatiga.
Anticuerpos anti-músculo liso positivos Hace 2 semanas se agregó ictericia. Acudió con
Anticuerpos anti-mitocondriales positivos múltiples médicos quienes dieron tratamiento de
Colangiorresonancia con vía biliar arrosariada soporte para “hepatitis viral aguda” sin mejoría. Hace
Se trata de un cuadro típico de colangitis biliar primaria. 95% de los pacientes
con esta entidad, tienen anticuerpos anti-mitocondriales positivos.
2 días inició con alteración del estado de alerta con
En caso de realizar biopsia hepática, ¿cuál es el somnolencia. Actualmente tiene encefalopatía grado
hallazgo esperable? 3. Los exámenes de laboratorio: BT 5 mg/dL, BD 1
Lesión ductal florida y ductopenia mg/dL, BI 4 mg/dL, ALT 100 U/L, AST 220 U/L, FA 30
Inflamación colangiolar en capas de cebolla U/L. BH con Hb de 11 g/dL, LDH 500 U/L, INR 3.
Hepatitis de interfase con células plasmáticas Esta entidad se produce por acumulación de:
Hepatitis lobulillar con colestasis intrahepática Hierro
Al ser un cuadro típico de colangitis biliar primaria, el hallazgo histológico Cobre
típico es lesión ductal florida con granulomas y ductopenia. La inflamación Esfingolípidos
colangiolar en capas de cebolla es de CEP y la hepatitis de interfase es de
hepatitis autoinmune. Alfa-1-antitripsina
Se trata de un caso de falla hepática aguda muy probablemente secundaria
¿Cuál es el tratamiento de elección en esta entidad que a enfermedad de Wilson (historia familiar de muerte por falla hepática aguda,
se ha asociado a mejoría del pronóstico? anemia hemolítica, fosfatasa alcalina baja, AST>ALT). La enfermedad de
Colestiramina Wilson se produce por acumulación de cobre.

Citalopram ¿Cuál de los siguientes genes se encuentra involucrado


Prednisona en la patogénesis de esta enfermedad?
Ácido ursodesoxicólico HFE
El ácido ursodesoxicólico es el tratamiento de elección en la CBP y los C282Y
pacientes que responden bioquímicamente tienen un mejor pronóstico que H63D
aquellos que no mejoran.
ATP7B
¿Cuál de los siguientes tratamientos se considera como Las otras opciones son alelos alterados en hemocromatosis y el gen HFE es la
primera elección para el tratamiento del prurito? principal causa de hemocromatosis.
Citalopram ¿Cuál de las siguientes manifestaciones sugiere otra
Colestiramina patología?
Ácido ursodesoxicólico Anemia hemolítica
Plasmaféresis Anillo de Kayser-Fleischer
El tratamiento inicial para los pacientes con prurito es la colestiramina. La Diabetes y cardiomiopatía por depósito
plasmaféresis se reserva para casos refractarios y en último lugar el
trasplante hepático para prurito intratable. Elevaciones leves de las transaminasas con fosfatasa
¿Cuál es la explicación de la aparición de manchas alcalina baja
La diabetes y cardiomiopatía son manifestaciones típicas de hemocromatosis
amarillentas en los párpados? (acúmulo de hierro) no por enfermedad de Wilson.
Acumulación de sales biliares ¿Cuál es el mejor tratamiento para este paciente?
Acumulación de colesterol Trientina
Vasculitis D-penicilamina
Acumulación de glucosaminoglucanos Trasplante hepático
Zinc Hepatocarcinoma fibrolamelar
La falla hepática aguda en pacientes con enfermedad de Wilson tiene Hepatocarcinoma
prácticamente 100% de mortalidad en ausencia de trasplante hepático. El
resto son tratamientos utilizados en pacientes diagnosticados con formas Colangiocarcinoma
crónicas de la enfermedad. Hiperplasia nodular focal
¿Cuál es la explicación más probable de las Se trata de una lesión que tiene comportamiento típico por imagen de
hepatocarcinoma lo cuál es diagnóstico en pacientes con cirrosis sin
manifestaciones neurológicas en su hermana? necesidad de obtener biopsia de la lesión.
Depósito de cobre en los ganglios basales Además se realizó estudio endoscópico, que mostró
Edema cerebral várices esofágicas grandes con datos de mal
Depósito de hierro en el núcleo lenticular pronóstico. Nunca ha tenido episodios de hemorragia.
Isquemia cerebral por vasculitis De acuerdo a esto, podríamos inferir que si a este
Los trastornos del movimiento en pacientes con enfermedad de Wilson son
secundarios a depósito de cobre en el núcleo lenticular. Es más frecuente paciente se le practicara un gradiente de presión
encontrar anillo de Kayser-Fleischer en pacientes con manifestaciones venosa hepática, ¿en cuál de los siguientes rangos de
neurológicas. presión se encontraría?
5-10 mmHg
CASO CLÍNICO 4 10-12 mmHg
Hombre de 60 años de edad con historia de ingesta de >12 mmHg
alcohol en exceso, suspendido hace 10 años (tomaba 15-20 mmHg
aproximadamente 80 gramos de alcohol al día). Se La presión portal normal es <5 mmHg. Entre 5 y 10 mmHg se considera
encuentra asintomático aunque tiene la duda de si hipertensión portal no clínicamente significativa. A partir de 10 mmHg
empieza la aparición de várices y después de 12 mmHg, se incrementa el
tiene cirrosis hepática por lo que acude para consulta riesgo de sangrado.
de rutina. ¿Cuál de las siguientes estrategias es la más apropiada
¿Cuál hallazgo descartaría cirrosis hepática? para prevenir un primer episodio de hemorragia
Trombocitopenia variceal?
Eritema palmar Metoprolol
Trombocitosis Propranolol
Telangiectasias en la mitad superior del cuerpo Ligadura + propranolol
La trombocitosis no es un hallazgo típico en un paciente con cirrosis hepática.
Ligadura + metoprolol
Debido a que la valoración clínica no fue concluyente, La ligadura y el tratamiento con beta bloqueador no selectivo son igualmente
usted decidió realizar un ultrasonido hepático que efectivos en la profilaxis primaria de hemorragia variceal.
mostró hígado macroscópicamente normal con los
bordes discretamente lobulados y bazo de tamaño CASO CLÍNICO 5
normal. Se realizó también elastografía hepática de Mujer de 60 años de edad previamente asintomática.
transición que reportó fibrosis “F4”. De acuerdo a . Acudió al servicio de urgencias por evacuaciones
estos hallazgos, ¿Qué estudio auxiliar es irrelevante en melénicas y sensación de desvanecimiento.
el seguimiento del paciente? Antecedente de transfusión de múltiples paquetes
Esofagogastroduodenoscopia globulares en 1980 por hemorragia obstétrica. En la
Ultrasonido hepático semestral exploración se documentó taquicárdica con FC de 110,
Inmunización para VHA y VHB TA 70/40mmHg y palidez de tegumentos. Los
Alfafetoproteína semestral exámenes de laboratorio muestran Hb 6.5 g/dL,
La alfafetoproteína no tiene ningún papel actualmente en el tamizaje de plaquetas 115 000, INR 1.2.
hepatocarcinoma por su baja sensibilidad.
¿Cuál es la conducta terapéutica más apropiada en
Pasados 6 meses, se solicitó estudio de imagen, el cuál
este momento?
mostró una lesión en lóbulo hepático derecho de 2 cm
Reanimación con cristaloides
de diámetro, con reforzamiento arterial y lavado en
Reanimación con coloides
fases tardías. De acuerdo a los hallazgos, ¿cuál de los
Iniciar terlipresina
siguientes es el diagnóstico diferencial más apropiado?
Realizar endoscopia emergente
La reanimación es el paso inicial en cualquier caso de hemorragia. Son de En un paciente con cirrosis y lesión renal aguda, antes de integrar el
elección los cristaloides. El resto del tratamiento puede iniciarse una vez diagnóstico de síndrome hepatorrenal deben corregirse causas de depleción
reanimado el paciente. de volumen (suspensión de diuréticos) y realizar expansión de volumen con
Como parte de su protocolo terapéutico se solicitaron albúmina. Si después de 48 horas de albúmina no hay mejoría y se descartan
causas secundarias (obstrucción urinaria, necrosis tubular aguda,
al banco de sangre paquetes globulares, ¿cuál es la glomerulopatías) puede integrarse el diagnóstico de síndrome hepatorrenal.
meta de hemoglobina en esta paciente? Durante la hospitalización, la paciente presentó
Hb 7-8 g/dL somnolencia, asterixis y rueda dentada. ¿Cuál de las
Hb >6-7 g/dL siguientes conductas no es apropiada en este
Hb >10 g/dL momento?
Hb normal Descartar desequilibrio electrolítico
Se prefieren estrategias de transfusión restrictivas. Al parecer cifras de
hemoglobina >8 confieren mayor riesgo de resangrado por lo que no debe
Revisar cuantas evacuaciones ha reportado enfermería
llevarse la hemoglobina a >8 g/dL. Iniciar lactulosa vía oral
Se realizó endoscopia, encontrando 3 paquetes de várices Realizar tomografía de cráneo
esofágicas grandes con puntos rojos. Una de ellas con un No es necesario realizar TAC de cráneo ya que la paciente no tiene
pezón blanquecino como estigma de sangrado reciente. Se antecedentes o datos que orienten a la presencia de alguna entidad
intracraneal.
realizó ligadura de las várices. Su evolución fue favorable y
fue egresada. Cuatro meses después acude por incremento
progresivo del perímetro abdominal y edema de miembros CASO CLÍNICO 6
inferiores. Niega fiebre o dolor abdominal. Exploración Mujer de 22 años de edad previamente sana. Tuvo
física: con matidez cambiante y ascitis grado 2. Se realizó hace 2 meses un episodio de colecistitis aguda que
paracentesis encontrando Albúmina de 1 g/dL (albúmina requirió colecistectomía resuelta sin complicaciones,
sérica de 3.5 g/dL), neutrófilos de 350 por campo, proteínas sin embargo en el ultrasonido se documentó de
totales de 1.2 g/dL. De acuerdo a estos hallazgos, ¿cuál es la forma incidental una lesión en lóbulo hepático
conducta a seguir? derecho de 2 cm de diámetro. No ha tenido síntomas,
Realizar tomografía de abdomen para buscar sin embargo se encuentra preocupada y acude a
perforación consulta.
Vigilancia ¿Cuál es la mejor conducta a seguir en este momento?
Ceftriaxona IV + Albúmina Vigilancia con ultrasonido semestral
Furosemida IV + albúmina Colangiorresonancia magnética
Se trata de una ascitis neutrocítica. Dicha entidad tiene 100% de mortalidad
sin tratamiento por lo que es mandatorio iniciar antibióticos (son de elección Tomografía trifásica de hígado
las cefalosporinas de tercera generación). Se recomienda además dar Cirugía con hepatectomía derecha
albúmina a 1.5 gramos por kg el día 1 y a 1 gramo por kg el día 2. El abordaje inicial de las lesiones hepáticas ya diagnosticadas mediante USG,
Su evolución fue favorable y egresó a los 5 días con consiste en estudios de imagen dinámicos en múltiples fases (arterial, portal,
venosa y tardía).
tratamiento a base de diuréticos. Dos semanas
La paciente acudió a seguimiento pero olvidó el
después, acudió nuevamente a urgencias por fatiga.
reporte. Solo recuerda que la hoja decía que tenía la
Los laboratorios reportaron Cr 2 mg/dL (0.8 hace dos
lesión hepática benigna más frecuente. ¿Cuál de los
semanas). Se realizó sedimento urinario que no
siguientes hallazgos se espera que el radiólogo haya
reportó alteraciones (blando), ultrasonido renal que
observado?
fue reportado normal. En el examen general de orina
Reforzamiento arterial y lavado venoso tardío
no hay proteinuria significativa. Se calculó la Fracción
Reforzamiento periférico y llenado centrípeto tardío
excretada de sodio (FeNa=0.1). ¿Cuál es la conducta
Reforzamiento venoso
más apropiada en este momento?
Lesión hepática rodeada de múltiples lesiones satélites
Suspensión de diuréticos, albúmina a 1 gr/kg por 48
e hígado nodular difuso
horas y revaloración La lesión benigna más frecuente de hígado es el hemangioma y el hallazgo
Iniciar albúmina y terlipresina IV típico es el reforzamiento periférico y llenado centrípeto tardío.
Iniciar albúmina y norepinefrina IV
Subir dosis de diuréticos para evitar sobrecarga hídrica
¿Cuál es la conducta más apropiada en este momento? ¿Cuál es la probabilidad de desarrollar complicaciones
Realizar biopsia y la probabilidad de recurrencia de episodios de dolor
Enviar a cirugía para resección similares a los que ha tenido?
Realizar resonancia con medio de contraste 10-15% complicaciones / 5-10% recurrencia de dolor
Vigilancia expectante 20-30% complicaciones / 30-40% recurrencia de dolor
El estudio de imagen razonablemente no da un diagnóstico y la biopsia de 2-3% complicaciones / 30-40% recurrencia del dolor
una lesión vascular confiere alto riesgo de sangrado por lo que por el
momento sólo requiere vigilancia. 5-6% complicaciones / 20-30% recurrencia del dolor
Una vez que un paciente tuvo episodios de cólico biliar tiene un riesgo de 2-
La paciente se quedó inquieta y acudió con otro 3% de desarrollar complicaciones y 30-40% de recurrencia de nuevos
médico quien realizó biopsia. ¿Cuál de los hallazgos episodios. La tasa de progresión de colelitiasis asintomática a sintomática es
histopatológicos es más probable en la entidad en de 2% por año por eso no está justificado el tratamiento quirúrgico como
primera opción.
cuestión?
La paciente decidió no operarse y acudió 3 meses
Cordones de hepatocitos maduros sin triadas portales
después por dolor en hipocondrio derecho, ictericia y
Espacios vasculares rodeados de células endoteliales
fiebre. Aun no desea operarse y solo requiere algún
con estroma fino
tratamiento que le resuelva el episodio actual.
Células poco diferenciadas en estructuras glandulares
Exploración física: taquicárdica. Los exámenes de
Hepatocitos que forman nódulos de regeneración sin
laboratorio muestran BT 4.5 mg/dL, BD 3.5 mg/dL, BI 1
fibrosis circundante
La presencia de cordones de hepatocitos maduros sin triadas portales es
mg/dL, FA 200 U/L. Leucocitos de 18000 x µL. El
sugerente de adenoma, la presencia de células poco diferenciadas que ultrasonido de hígado y vías biliares muestra dilatación
forman estructuras glandulares es sugestiva de metástasis hepáticas de de VB hasta 8 mm. ¿Cuál es la mejor conducta
adenocarcinoma y la presencia de hepatocitos que forman nódulos de
regeneración sin fibrosis es sugerente de hiperplasia nodular regenerativa. terapéutica en este momento?
La paciente toma anticonceptivos y desea Colangiorresonancia y si es positiva CPRE
embarazarse. ¿Cuál de las siguientes opciones es la CPRE desde el inicio
más apropiada respecto al uso de hormonales en esta Ultrasonido endoscópico y si es positivo CPRE
paciente? Colecistectomía con exploración de la vía biliar
La paciente tiene dos predictores muy fuertes de coledocolitiasis (colangitis
Existe contraindicación absoluta para su embarazo y BT >4 mg/dL) por lo que la probabilidad es alta. En estos casos el
Puede continuar anticonceptivos hasta que se tratamiento de elección es CPRE. Actualmente se considera la CPRE como el
embarace y el embarazo es de riesgo estándar estándar de tratamiento de litiasis de la VB antes de la colecistectomía por
su menor invasividad.
Debe resecarse la lesión antes del embarazo
Fue tratada exitosamente por su grupo médico y fue
Se trata de un embarazo de alto riesgo y de preferencia
egresada, sin embargo, 6 meses después, acude
esperar 2 a 3 años para planearlo
El único tumor hepático que requiere cambio respecto a la conducta en el
nuevamente al servicio de urgencias por dolor en
manejo de hormonales y/o embarazo es el adenoma, pero no es el caso. hipocondrio derecho de 8 horas de evolución, intenso,
acompañado de fiebre en las últimas 2 horas. Se realizó
Colelitiasis / Pancreatitis aguda y crónica / Tumores ultrasonido de hígado y vías biliares que mostró
de páncreas vesícula biliar distendida con engrosamiento de su
CASO CLÍNICO 1 pared de 5 mm y signo de Murphy ultrasonográfico.
Mujer de 40 años de edad previamente sana. Inició ¿Cuál es la fisiopatogenia del cuadro clínico actual de
hace un año con dolor en hipocondrio derecho la paciente?
postprandial intermitente (aproximadamente un Obstrucción del colédoco por lito
episodio al mes), intenso, con irradiación a escápula Obstrucción transitoria del cístico por lito
derecha y acompañado de náusea y ocasionalmente Obstrucción persistente del cístico por lito
vómito de contenido gástrico. La paciente no desea Infección por anaerobios productores de gas
operarse. La fisiopatología es obstrucción persistente del cístico por un lito y en algunos
casos infección bacteriana secundaria. Cuando la vesícula se infecta por
anaerobios productores de gas, se produce colecistitis enfisematosa lo cual
es poco frecuente aunque grave.
Durante la cirugía se encuentra vesícula biliar friable después de 48 a 72 horas y cuando se descartaron causas comunes mediante
el USG y exámenes de laboratorio.
con múltiples adherencias circundantes y una fístula
¿Cuál de las siguientes es la forma más apropiada para
colecistoduodenal. Se realiza cierre primario y
reanimar a la paciente?
colecistectomía sin complicaciones. ¿Cuál de las
Solución glucosada al 5% 5-10 mL/Kg/hr
siguientes entidades se asocia a la presencia de una
Solución mixta 5-10 mL/Kg/hr
fístula colecistoduodenal?
Solución salina al 0.9% 5-10 mL/kg/hr
Síndrome de Mirizzi
Solución Ringer lactato 5-10 mL/kg/hr
Colecistitis enfisematosa La solución de elección es Ringer lactato hasta lograr las metas de resolución.
Íleo biliar El requerimiento promedio de volumen en un paciente con estas
Coledocolitiasis características es de 2.5 a 4L. Hay que llevar el hematocrito a 44%.
Para que sea posible una oclusión intestinal por litos, es necesario que exista La paciente ha evolucionado favorablemente después
una fístula colecistoduodenal ya que los litos obstructivos usualmente son de 48 horas de estancia hospitalaria. Actualmente ya
grandes (2.5 cm) y no pueden pasar por el colédoco. Los sitios más frecuentes
de obstrucción son el íleon terminal y la salida gástrica, este último llamado
no tiene dolor. ¿Cuál es la conducta más apropiada en
síndrome de Bouveret. este momento?
En el postoperatorio, la paciente presentó fiebre, dolor Iniciar nutrición enteral
e ictericia. Los exámenes de laboratorio muestran BD Iniciar vía oral
3.5 mg/dL, FA 300 U/L y el ultrasonido muestra Iniciar nutrición parenteral
dilatación de la vía biliar hasta 10 mm. La paciente no Iniciar vía oral previa medición de enzimas pancreáticas
deseó ningún tratamiento adicional y 24 horas Se trata de un caso de pancreatitis aguda leve. Este grupo de pacientes puede
ser alimentado por vía oral en cuanto se resuelva el dolor.
después, inició con deterioro del estado de alerta e
Este hallazgo te hablaría de complicación de la
inestabilidad hemodinámica. Los exámenes de
paciente:
laboratorio muestran BT 6 mg/dL, BD 5 mg/dL,
Edema pancreático en la tomografía
leucocitos de 25000 x µL. ¿Cuál es la mejor conducta a
Creatinina de 0.8 a las 48 horas
seguir?
Índice de Kirby de >400 a las 48 horas
Cirugía urgente
Signo de Grey-Turner
CPRE urgente El signo de Grey-Turner es un dato de pancreatitis necrótico-hemorrágica por
Drenaje biliar percutáneo lo que esperaríamos ver a un paciente con falla orgánica persistente y otros
parámetros de gravedad. El resto puede verse en pacientes con pancreatitis
Antibióticos y tratamiento conservador
leve como en este caso.
Se trata de una colangitis grave con Péntada de Reynolds que requiere
drenaje urgente de la vía biliar. El método de elección es CPRE. El drenaje Se solicitó un estudio de imagen que mostró la
biliar percutáneo se reserva para los casos de CPRE fallida. presencia de múltiples litos en la vesícula biliar. No hay
dilatación de la vía biliar. Las pruebas de función
CASO CLÍNICO 2 hepática son normales. ¿Cuál es la mejor conducta a
Mujer de 40 años de edad, previamente sana y sin seguir en esta paciente?
antecedentes de importancia. Acude al servicio de CPRE con esfinterotomía y colecistectomía electiva
urgencias por dolor abdominal en epigastrio, intenso, CPRE y colecistectomía en esta hospitalización
transfictivo con irradiación en hemicinturón hacia la Colecistectomía durante esta hospitalización
espalda. Los exámenes de laboratorio son normales Ultrasonido endoscópico y si es positivo, CPRE con
con excepción de la lipasa, que se reporta en 1100 U/L. posterior colecistectomía durante hospitalización
Esta indicación la aplicas si has descartado las causas A pesar de que las PFH son normales y no hay dilatación de la vía biliar, la
más comunes: paciente tiene un predictor moderado de colédoco-litiasis (pancreatitis
biliar). Lo ideal es descartar coledocolitiasis mediante un método no invasivo
Hidratación vigorosa (colangiorresonancia o ultrasonido endoscópico) y CPRE sólo en caso de ser
Realizar ultrasonido de hígado y vías biliares positivo. Por otro lado, en pacientes con pancreatitis leve, se recomienda
colecistectomía antes del alta.
Realizar tomografía de abdomen
Ayuno y analgesia
En este caso se trata de un cuadro típico por lo que el papel de la tomografía
en estos casos es limitado. Debe reservarse para pacientes que no mejoran
CASO CLÍNICO 3 La definición actual de pancreatitis aguda grave es la presencia de falla
orgánica persistente por >48 horas de acuerdo a los criterios de Marshall. La
Hombre de 60 años de edad diabético y con obesidad falla orgánica transitoria y/o la presencia de complicaciones locales o
mórbida. Inició hace 2 horas con dolor abdominal exacerbaciones de comorbilidades previas definen a la pancreatitis
epigástrico, transfictivo con irradiación hacia moderada.

hipocondrio izquierdo por lo que acudió al servicio de ¿Cuál es la mejor forma de iniciar el tratamiento
urgencias. Exploración física: paciente taquicárdico y nutricional en este paciente?
con fiebre (38.5°C). La saturación de oxígeno al aire Mantener sin aporte calórico por los primeros 7 días y
ambiente es de 85% por lo que se pasa a área de después revalorar
hospitalización y se inicia oxígeno con puntas nasales Nutrición parenteral total
con corrección de la hipoxemia. Los exámenes de Nutrición enteral
laboratorio muestran BT 5 mg/dL, BD 4 mg/dL, BI 1 Vía oral
La nutrición enteral temprana se asocia con mejores desenlaces en
mg/dL, FA 200 U/L, ALT 200 U/L, AST 185 U/L. Amilasa pancreatitis aguda. La nutrición parenteral se reserva para los casos de pobre
1000 U/L, triglicéridos 500 mg/dL. Calcio corregido de tolerancia o bien cuando hay alguna condición que impida el uso del tracto
digestivo.
11 mg/dL.
El paciente ha evolucionado favorablemente, sin
¿Cuál es la causa más probable de la pancreatitis aguda
embargo, hace 1 día, se realizó tomografía de
en este paciente?
abdomen que mostró necrosis no encapsulada de 30%
Hipertrigliceridemia
del parénquima pancreático hacia el cuerpo y cola sin
Biliar
gas. El paciente no ha tenido datos de respuesta
Alcohol
inflamatoria sistémica ¿Cuál es la mejor conducta en
Hipercalcemia
La hipertransaminasemia y el resto de alteraciones de las PFH son sugestivas este momento respecto a la necrosis?
de pancreatitis biliar. El nivel de triglicéridos descrito para pancreatitis es Drenaje percutáneo de la necrosis
>1000 mg/dL. El paciente niega ingesta de alcohol y no hay algún otro dato Vigilancia expectante
orientador al respecto.
Dos horas después, inicia además con dolor en Punción de la necrosis e iniciar antibióticos si es positiva
hipocondrio derecho y nuevo pico febril acompañado de Iniciar antibióticos profilácticos
En pacientes con pancreatitis aguda y colecciones se prefiere el tratamiento
escalofríos. Se documenta hipotensión con TA de más conservador posible, en especial cuando son colecciones tempranas.
80/50mmHg que responde a líquidos. Además de
continuar con la reanimación, ¿cuál es la conducta más CASO CLÍNICO 4
apropiada en este momento? Hombre de 55 años, ex alcohólico actualmente. Inició
Antibióticos y vigilancia hace 2 meses con diarrea con esteatorrea y lientería en
CPRE y antibióticos abundante cantidad, acompañada de pérdida de 5 kg
Iniciar aminas de peso durante este período. Ha tenido episodios de
Realizar TAC y normar conducta a seguir de acuerdo a dolor epigástrico transfictivo de moderada intensidad
los hallazgos que lo incapacitan de sus actividades cotidianas.
Se trata de una pancreatitis aguda con colangitis, que es una de las pocas
indicaciones de CPRE temprana en pacientes con pancreatitis aguda (la otra Solicitaste estudio de imagen que mostró páncreas
indicación es colestasis persistente). atrófico con múltiples calcificaciones y litos
Se lleva a cabo el manejo que usted indicó, sin embargo intraductales con dilatación del conducto pancreático
el paciente persiste con datos de respuesta inflamatoria hasta 7 mm.
y a las 48 horas presenta hipoxemia refractaria que ¿Cuál es la causa más frecuente de la enfermedad de
requiere intubación endotraqueal. ¿Cuál de los siguientes este paciente?
parámetros clasificaría a este paciente como pancreatitis
Biliar
aguda grave?
Hipertrigliceridemia
Hipoxemia persistente
Tabaquismo
Hipoxemia refractaria
Alcohol
Obesidad mórbida y episodio de hipotensión transitorio La causa más frecuente de pancreatitis crónica es alcohol.
Hipotensión transitoria
La patología citada no es un factor de riesgo para: para control del dolor pero a largo plazo, la efectividad del tratamiento
quirúrgico es mayor.
Adenocarcinoma de páncreas
Obstrucción de la vía biliar
CASO CLÍNICO 5
Obstrucción del duodeno
Mujer de 60 años de edad. Hace 1 mes acudió a una
Cistadenomas serosos de páncreas
La pancreatitis crónica no es un factor de riesgo para desarrollo de
revisión con su gastroenterólogo, quien solicitó una
cistadenomas serosos de páncreas. Como complicaciones puede ocasionar tomografía abdominal, encontrando una lesión
obstrucción del biliar y duodenal y en un porcentaje pequeño, quística en la cabeza del páncreas de 2 cm de diámetro
adenocarcinoma de páncreas.
con patrón en panal de abejas. La describen como una
¿Cuál es el tratamiento inicial recomendado para la
lesión lobulada con múltiples microquistes en su
diarrea malabsortiva en este paciente?
interior con una cicatriz central. No hay dilatación del
Descompresión del conducto pancreático
conducto pancreático ni de la vía biliar.
Enzimas pancreáticas sin cubierta entérica
¿Cuál es el siguiente paso en el abordaje
Enzimas pancreáticas con cubierta entérica
diagnóstico/terapéutico en esta paciente?
Antibióticos
El tratamiento inicial de elección para la insuficiencia exocrina es lipasa Ultrasonido endoscópico con biopsia
pancreática, por lo menos 30000 U diarias. Pancreatografía endoscópica
El paciente ha mantenido seguimiento en la consulta Resección de la lesión
externa, sin embargo en los últimos meses ha tenido Vigilancia expectante
aumento progresivo del dolor. ¿Cuál efecto adverso no Con el estudio de imagen realizado, queda claro que se trata de un
cistadenoma seroso de páncreas el cual tiene un extraordinariamente bajo
se relaciona con el tratamiento médico de elección en potencial maligno. Se encuentra en una localización típica de este tipo de
el control a largo plazo del dolor moderado a grave en tumores (cabeza), su morfología por imagen es típica (microquistes y cicatriz
pancreatitis crónica? central), no hay síntomas y no hay datos de obstrucción por lo que la paciente
puede ser vigilada sin necesidad de otro estudio diagnóstico.
Constipación
¿Cuál es el potencial de transformación maligna de la
Somnolencia
neoplasia quística en esta paciente?
Úlcera péptica
0-1%
Dependencia
Los efectos adversos son dismotilidad intestinal (constipación, retraso en el
5%
vaciamiento gástrico), dependiendo del opiáceo los efectos cognitivos son 10%
variables pero pueden causar somnolencia y finalmente, el efecto más 30%
indeseable que limita el uso de opiáceos en pancreatitis crónica es el El potencial maligno es prácticamente nulo y esta es una de las razones por
desarrollo de tolerancia/dependencia y adicción a opiáceos en algunos las que se recomienda tratamiento expectante.
pacientes.
La hermana de la paciente de 52 años, se quedó
Se optimizó el tratamiento médico del paciente, sin
preocupada y ella también se realizó una tomografía
embargo continúa con dolor. Se realizó nueva
que mostró una lesión quística en la unión del cuerpo
tomografía, encontrando los litos intraductales
del páncreas con la cola. Dicha lesión tiene un
previamente observados con dilatación del conducto
diámetro de 3 cm, es de aspecto macroquístico y tiene
pancreático a 8 mm sin otras alteraciones. Con esta
calcificación periférica. Al interrogatorio refiere que se
información, de los siguientes, ¿Cuál es el mejor
encuentra asintomática. ¿Cuál de las siguientes
tratamiento a ofrecer en este momento?
opciones es característica exclusiva en esta lesión?
Pancreatoduodenectomía (Whipple)
Tiene potencial maligno y debe resecarse
Pancreatectomía-total
Se comunica con el conducto pancreático
Pancreatectomía distal con esplenectomía
Solo las mujeres son afectadas por esto tumores
Pancreatoyeyuno anastomosis (Puestow)
En general en pancreatitis crónica, siempre se prefiere conservar la mayor Su principal característica histológica es la presencia de
parte de parénquima pancreático. Los pacientes con dilatación del conducto estroma ovárico
pancreático son los mejores candidatos a procedimientos derivativos ya que Se trata de un tumor mucinoso que tiene aspecto típico por imagen
su anatomía permite hacer la pancreatoyeyuno anastomosis laterolateral (macroquístico con calcificación periférica en cáscara de huevo). Este grupo
con descompresión del parénquima pancreático. Se ha comparado de tumores son casi exclusivos de mujeres.
tratamiento quirúrgico vs endoscópico y a corto plazo tienen eficacia similar
La paciente es médico y no desea tratarse e insiste en ¿Cuál de los siguientes se desconoce como un factor de
la realización de un ultrasonido endoscópico con riesgo para el desarrollo de este tipo de neoplasias?
aspiración. ¿Cuál de los siguientes hallazgos es Pancreatitis crónica
característico en este tipo de lesiones? Tabaquismo
Elevación del antígeno carcinoembrionario en el Neoplasia intraductal papilar mucinosa
líquido Edad 20-30 años
Elevación del Ca-19-9 en el líquido Estos tumores típicamente se observan en pacientes de edad avanzada.

Elevación de la amilasa en el líquido ¿Cuál de las siguientes no es una característica de este


Mucina ausente tipo de neoplasias?
El marcador de neoplasias mucinosas en el líquido de aspiración es el La mayoría son resecables al diagnóstico
antígeno carcinoembrionario. Niveles altos sugiere una neoplasia mucinosa. Tienen una alta tasa de mortalidad
Se realiza el tratamiento indicado y permanece El tipo más frecuente es el adenocarcinoma ductal
asintomática por lo que no acudió ya a más consultas Los tumores de la cabeza causan obstrucción de la vía
de seguimiento. Regresa con usted 10 años después biliar
debido a que en una tomografía realizada le La mayoría de estos tumores se presentan en etapas avanzadas al diagnóstico
detectaron dilatación del conducto pancreático de 5 y son irresecables, sólo 10 a 20% son resecables. El adenocarcinoma ductal
(el tipo más frecuente) es uno de los cánceres más letales. Los tumores de la
mm. Le solicita una pancreatografía por resonancia cabeza tienden a presentarse antes que los del cuerpo y cola debido a que
magnética que mostró una lesión de 1 cm de diámetro causan ictericia y dilatación de la vía biliar.
en el conducto pancreático principal, en la cabeza del Usted decidió solicitar una tomografía trifásica de
páncreas. ¿Cuál de las siguientes no es una páncreas, la cual reportó una tumoración en la cabeza
característica típica de este tipo de lesiones? del páncreas de 2.2 cm con signo del doble conducto.
Producen moco La tumoración pierde interfase con la arteria
Tienen potencial maligno y deben resecarse mesentérica superior en la mayor parte de su
Pueden producir episodios de pancreatitis aguda circunferencia. Se realizó ultrasonido endoscópico con
Tienen niveles bajos de antígeno carcinoembrionario biopsia que mostró glándulas atípicas con invasión del
en el líquido estroma y abundante reacción desmoplásica. De
Se trata de una neoplasia papilar intraductal mucinosa en inglés IPMN. Al ser acuerdo a estas características, ¿cuál es el mejor
neoplasias mucinosas, se espera un contenido elevado de ACE en el líquido.
Debido a obstrucción del sistema ductal, tienen el potencial de presentarse
tratamiento en este momento?
con episodios de pancreatitis aguda. Por su potencial maligno deben Cirugía de Whipple
resecarse. Colocación de prótesis biliar y posteriormente cirugía de
Whipple
CASO CLÍNICO 6 Quimioterapia
Hombre de 70 años con historia de consumo excesivo Neurolisis del plexo celiaco
de alcohol y tabaquismo con IT 30. (Ambos Se trata de una lesión irresecable en la cabeza del páncreas que tiene
actualmente suspendidos). Inició hace 3 meses con confirmación histológica de adenocarcinoma por lo tanto no puede ofrecerse
ninguna opción quirúrgica. En estos casos el tratamiento a seguir es
dolor epigástrico intermitente, tolerable con quimioterapia/paliación de síntomas. El tratamiento del dolor inicialmente
irradiación a hipocondrio izquierdo. Al interrogatorio es médico.
dirigido refiere pérdida de 12 kg de peso desde el inicio ¿Qué fármacos se encuentra fuera de los
de su padecimiento. Se indicó realización de quimioterapéuticos utilizados para este tipo de
tomografía de abdomen que mostró una lesión mal neoplasias?
definida en la cabeza del páncreas de 2 cm con Gemcitabina
dilatación del conducto pancreático de 5 mm, Oxaliplatino
dilatación del colédoco y páncreas atrófico en cuerpo Fluoracilo
y cola. L-asparaginasa
Las principales quimioterapias utilizadas son la gemcitabina y el esquema
FOLFIRINOX (Ácido Folínico, Fluoracilo, Irinotecan, Oxaliplatino). La L-
asparaginasa no tiene ningún papel.
En el caso de tumores resecables que se encuentran en Dysplasia
la cabeza del páncreas, ¿Cuál es el procedimiento de None of the above
elección? With endoscopic view, one can have a general picture if the patient has or no
Barrett esophagus but definitive diagnosis is made by histology.
Pancreatoduodenectomía
In addition to the main finding that you selected in the
Resección de la cabeza del páncreas
previous question, which of the following histologic
Pancreatectomía y esplenectomía
findings is mandatory for the diagnosis of Barrett’s
Pancreatectomía distal
El estándar de tratamiento quirúrgico estándar para los tumores de la cabeza
esophagus?
de páncreas, es la pancreatoduodenectomía (Cirugía de Whipple). Dicha Presence of globet cells
cirugía puede o no preservar el píloro y requiere reconstrucción de la vía biliar Heterotopic gastric mucosa
(hepato-yeyuno anastomosis), anastomosis del páncreas residual con el
intestino (pancreatoyeyuno anastomosis) y Reconstrucción del tubo Ectopic pancreatic tissue
digestivo (gastroyeyuno o duodeno-yeyuno anastomosis). Lymphoid infiltrates
PMN infiltrates
The presence of globet cells is mandatory for the pathologist for the
FINAL definitive diagnosis.
CLINICAL CASE 1 Multiples biopsies revealed Barrett’s esophagus
A 60-year-old man is referred with a 10-year history of without dysplasia. Then you decide:
gastro esophageal reflux for which he has been taking Endoscopy surveillance
pantoprazol 40 mg orally 30 minutes before breakfast PPI therapy for symptoms
every day. He has been asymptomatic since he began Sucralfate
the PPI therapy. He smokes 4 cigarettes per day since A and B are correct
he was 20. B and C are correct
Patients with Barrett, need long term endoscopic surveillance. The
What is the next step in the management of this periodicity of this depends on the presence or absence of dysplasia. Patients
patient? with dysplasia need more aggressive surveillance. Also this patients need life-
Continue PPI therapy with the same dose long PPI therapy.

Perform an upper endoscopy


Perform and upper endoscopy only if the patient CASO CLÍNICO 2
develops alarm symptoms Mujer de 70 años de edad con hipertensión arterial
Performa an ambulatory 24-hour pH monitoring sistémica en tratamiento con amlodipino. Desde hace 6
Consider surgical treatment for gastro esophageal reflux meses presenta dolor abdominal epigástrico, plenitud
It’s a patient with multiple risk factors for Barrett esophagus. In general temprana, edema de miembros inferiores. Sus
there’s no role for Barrett’s screening except for men older than 50 years laboratorios con Hb 13 g/dl, Cr 1 mg/dl, albúmina 2 g/dl,
with more than 5 years of GERD which are obese or smoke. The presence of
this factors are all indications for screening. EGO sin proteinuria. Se le realizó una panendoscopia en
The following are risk factors for developing Barrett’s la que se observó engrosamiento de pliegues gástricos.
esophagus, except: ¿Cuál es el diagnóstico más probable?
Men Adenocarcinoma gástrico
Age >50 years Linfoma gástrico
Tobacco Síndrome de Zollinger-Ellison
Alcohol Gastroenteritis eosinofílica
BMI Enfermedad de Ménétrier
As previously stated all of the listed factors in the options are risk factors El engrosamiento de pliegues gástricos aunado a la hipoalbuminemia de
except alcohol. origen no renal sugiere una enteropatía perdedora de proteínas.
Which of the following make the diagnosis of Barrett`s Con respecto a la hipoalbuminemia de esta paciente,
esophagus: es cierto que:
Intestinal metaplasia Es secundaria a una enteropatía perdedora de proteínas
Atrophy Es por pérdidas renales y amerita recolección de orina
Ulcer de 24 horas
No tiene relación con la sospecha clínica Usted quiere saber si se logró la erradicación de H.
B y C son correctas pylori. ¿Qué estudio o estudios se pueden utilizar para
Tiene ego sin proteinuria y los hallazgos endoscópicos correlacionan con una corroborar esto?
probable enfermedad de Ménétrier.
Serología para Helicobacter
¿Cuál de los siguientes es un diagnóstico diferencial?
Prueba de aliento
Cáncer gástrico
Panendoscopia con toma de biopsias
Síndrome de Zollinger-Ellison
A y B son correctas
Sarcoidosis
B y C son correctas
Gastropatía hipertrófica asociada a H. pylori Tanto la prueba de aliento como cultivo son de utilidad para corroborar
Todas las anteriores erradicación de H. pylori. Se empieza por el menos invasivo. Sin embargo la
Todas las anteriores dan engrosamiento de pliegues gástricos similar a pregunta está dirigida a todos los métodos para corroborar erradicación.
enfermedad de Ménétrier. ¿Cuál sería el riesgo de metaplasia intestinal
El diagnóstico definitivo de esta patología es: incompleta a largo plazo?
Hallazgos endoscópicos No existe ningún riesgo
Biopsia de pared gástrica Úlcera péptica
Por la hipoalbuminemia Perforación gástrica
Diagnóstico de exclusión Cáncer gástrico
Ninguna de las anteriores B y C son correctas
La histología es indispensable para realizar este diagnóstico. Metaplasia intestinal incompleta con sulfocianina + es el que mayor riesgo de
Tratamiento de esta patología es: cáncer gástrico.
Gastrectomía ¿Qué tipo de metaplasia es la que tiene mayor riesgo
Vigilancia endoscópica de cáncer gástrico?
IBP Metaplasia intestinal completa con sialomucina +
No amerita tratamiento Metaplasia intestinal incompleta con sialomucina -
A y B son correctas Las dos presentan el mismo riesgo
Tratamiento con gastrectomía por riesgo asociado de cáncer gástrico. Sin La metaplasia con sialomucinas +
embargo se puede optar por vigilancia endoscópica regular.
Metaplasia intestinal incompleta con sulfocianina +
Metaplasia intestinal incompleta con sulfocianina + es el que mayor riesgo de
CASO CLÍNICO 3 cáncer gástrico.
Paciente masculino con antecedente de úlcera péptica En el seguimiento de este paciente será necesario:
asociada a H. pylori documentada hace 2 meses, recibió Realizar panendoscopia de vigilancia
tratamiento de erradicación. Se le tomaron biopsias No amerita seguimiento si se corrobora erradicación de
gástricas que reportaron metaplasia intestinal H. pylori
incompleta en cuerpo y antro sin displasia. Actualmente El pepsinógeno sirve como marcador tumoral
se encuentra asintomático. A y C son correctas
¿Cuál de las siguientes es una indicación de Ninguna
erradicación de Helicobacter pylori? Tiene metaplasia intestinal incompleta y extensa (afecta dos áreas del
estómago) por lo que amerita vigilancia por riesgo incrementado de
Dispepsia adenocarcinoma.
Páncreas ectópico en estómago
Linfoma gástrico y úlcera péptica CASO CLÍNICO 4
Todas las anteriores Masculino de 20 años de edad con DM1 sin
A y C son correctas complicaciones crónicas. Acude a consulta externa para
La indicaciones de erradicación más frecuentes son cáncer gástrico
temprano, linfoma gastrointestinal, úlcera gástrica, úlcera duodenal y
abordaje de diarrea crónica compatible con
dispepsia. malabsorción intestinal en base a cuantificación de
grasas en 24 hr. La prueba de D-xylosa por debajo de los
rangos adecuados.
Todas las siguientes puede ser causas de diarrea
crónica en el paciente, excepto: CASO CLÍNICO 5
Fibrosis quística Hombre de 55 años de edad con antecedente de ingesta
Enfermedad Whipple excesiva de alcohol. Refiere que toma
Enfermedad de Crohn aproximadamente 375 mL de ron al día desde hace
Enfermedad celiaca aproximadamente 20 años. Acude al servicio de
Sobrepoblación bacteriana urgencias por ictericia. De acuerdo a estos datos.
Se trata de una diarrea malabsortiva originada en la pared intestinal por lo ¿Qué cantidad de alcohol en gramos toma
que los diagnósticos diferenciales solo deben incluir condiciones originadas
en este sitio. La sobrepoblación bacteriana es una causa intraluminal sin diariamente?
embargo ocasiona disminución de la cifra de D-xilosa. 60 gramos
Su sospecha diagnóstica es: 40 gramos
Enfermedad celiaca 70 gramos
Sobrepoblación bacteriana 90 gramos
Insuficiencia pancreática exocrina 110 gramos
Diarrea por sales biliares La cantidad ingerida de alcohol en gramos se obtiene al multiplicar los mL de
la bebida consumida (en este caso ron) por su porcentaje de alcohol
Ninguna de las anteriores (usualmente 0.38) por 0.78 (un mL de alcohol pesa 1 gramo).
Usualmente la enfermedad celíaca se asocia a otras condiciones
autoinmunes, entre ellas DM1. Los pacientes con DM de larga evolución ¿Cuál de las siguientes opciones representa mejor la
pueden tener sobrepoblación bacteriana por dismotilidad intestinal, en este cantidad diaria máxima recomendada de consumo de
paciente es poco probable ya que no tiene complicaciones crónicas.
alcohol?
Ante la sospecha de enfermedad celiaca usted solicita 20 gramos de alcohol en hombres / 30 gramos de
inmunoglobulinas que fueron normales, el estudio de alcohol en mujeres
laboratorio con mayor rendimiento diagnóstico es: 30 gramos de alcohol en ambos sexos
Anticuerpo antigliadina IgG 60 gramos de alcohol en hombres / 30 gramos de
Anticuerpo antigliadina IgA alcohol en mujeres
Anticuerpo antiendosmisio IgG 30 gramos de alcohol en hombres / 20 gramos de
Anticuerpo antitransglutaminasa por IgA alcohol en mujeres
Anticuerpo antitransglutaminasa IgG 10 gramos de alcohol en hombres / 20 gramos de
Los anticuerpos antitransglutaminasa IgA son de elección para el tamizaje de
celíaca. Un 5% tienen deficiencia de IgA y en ellos la prueba de elección son alcohol en mujeres
los antigliadina IgG. La opción resaltada representa la cantidad diaria máxima recomendada de
El resultado fue positivo. Esta enfermedad se asocia al ingesta de alcohol. El hombre tiene una tolerancia mayor al daño hepático
por alcohol que la mujer. A partir de 60 gramos de alcohol diario en hombres
siguiente HLA: y aproximadamente 30-40g en mujeres, la presencia de esteatohepatitis
DQ2 alcohólica se eleva notablemente.
DR4
DQ6 A la EF se encuentra ictérico con hipertrofia parotídea.
DR8 Se solicitaron exámenes de laboratorio que mostraron
Ninguno de los anteriores BT de 22 mg/dL, AST 200 U/L, ALT 100 U/L, GGT 500
Los anticuerpos DQ2 y DQ8 se asocian a celíaca, y su ausencia tiene un valor U/L, TP 22/12, creatinina 0.9 mg/Dl, Hb de 10 g/dL con
predictivo negativo muy alto para descartar esta enfermedad. VGM 69 fL, plaquetas 140 x 103. ¿Cuál de los hallazgos
En la biopsia de duodeno usted espera encontrar lo encontrados en el paciente NO es explicable por su
siguiente, excepto. ingesta excesiva de alcohol?
Infiltrado intraepitelial por linfocitos Patrón de ALT/AST 1:2
Atrofia de vellosidades Elevación de la GGT a 500
Aplanamiento de vellosidades Hipertrofia parotídea
Infiltrado por polimorfonucleares Hiperbilirrubinemia de 22
Hiperplasia de criptas VGM 69
El infiltrado típico en enfermedad celíaca es linfocítico.
En la presencia de ingesta excesiva de alcohol, se espera la presencia de Se trata de una paciente con dolor abdominal crónico, historia de sangrado
macrocitosis. Cuando hay microcitosis como en este caso es mandatorio de tubo digestivo y elevación de marcadores inflamatorios por lo que el
descartar causas de deficiencia de hierro (principalmente sangrado). primer diagnóstico diferencial es enfermedad inflamatoria intestinal. La
¿Cuál de los siguientes hallazgos histológicos no es enfermedad de Crohn es más probable en este caso debido a la afección
segmentaria del íleon terminal.
probable encontrar en caso de que a este paciente se
De acuerdo a su diagnóstico presuncional, ¿cuál es el
le realizara una biopsia hepática?
siguiente paso en el abordaje diagnóstico?
Megamitocondrias
Colonoscopia con ileoscopia
Cuerpos de Mallory
Enteroscopia retrógrada
Infiltrado con polimorfonucleares
Videocápsula endoscópica
Esteatosis macrovesicular
Coprocultivo para gérmenes poco usuales
Hepatitis de interfase
Todas las opciones enlistadas son hallazgos característicos de las
Entero Resonancia magnética
esteatohepatitis. Hay que recordar que a menos que el infiltrado neutrofílico Se trata de un cuadro con alta sospecha de enfermedad de Crohn.
sea muy intenso, al patólogo le es imposible distinguir entre esteatohepatitis Recordando que la forma más frecuente de enfermedad de Crohn es la
alcohólica y no alcohólica. La hepatitis interfase es un hallazgo típico de ileocolónica, el siguiente paso es colonoscopía con ileoscopia con toma de
hepatitis autoinmune. biopsias. Si bien el resto de estudios tiene cierto papel en algún punto del
abordaje de la enfermedad de Crohn, no todos los pacientes los requieren.
Se calculó un puntaje de Maddrey de 68. De acuerdo a
este valor, ¿cuál es la mejor conducta a seguir con este
¿Cuál de las siguientes no es una característica de esta
paciente?
enfermedad?
Pentoxifilina
Úlceras lineales profundas con áreas de mucosa sana
Prednisona
intercaladas con zonas de mucosa enferma
Pentoxifilina + Prednisona
Afección universal del recto
Infliximab
Elevación de títulos de ASCAS en algunos pacientes.
Plasmaféresis
El tratamiento de elección en la hepatitis alcohólica grave (Maddrey >32) son
Afección perianal en algunos pacientes
los esteroides. Existen contraindicaciones para el uso de esteroides como Fístulas enterales en algunos pacientes
hemorragia de tubo digestivo activa, infecciones graves, falla renal y en estos La afección universal del recto es característica de CUCI no de Crohn. El resto
casos una alternativa es la pentoxifilina. La combinación de esteroides con de opciones enlistadas, corresponden a enfermedad de Crohn.
pentoxifilina no tiene ningún papel. ¿Cuál de los siguientes fármacos no es útil en el
tratamiento a largo plazo de esta enfermedad?
CASO CLÍNICO 6 Azatioprina
Mujer de 23 años sin antecedentes de importancia. 6-mercaptopurina
Acude a consulta debido a dolor abdominal episódico Prednisona
inespecífico de 2 años de evolución. El dolor es casi Infliximab
siempre tipo cólico en mesogastrio, acompañado de Metotrexate
distensión abdominal sin relación con las evacuaciones. Los esteroides son útiles en la inducción de la remisión en enfermedad
En dos ocasiones ha tenido evacuaciones con inflamatoria intestinal, no para el tratamiento de mantenimiento.

hematoquezia en cantidad moderada. En los exámenes ¿Cuál de las siguientes no es una manifestación
de laboratorio destaca Hb 11 g/dL, VGM 85, VSG 50 extraintestinal típica de esta enfermedad?
mm/h. Se realizó una tomografía de abdomen que Eritema nodoso
mostró engrosamiento concéntrico del íleon terminal y Pioderma gangrenoso
la válvula ileocecal, coprocultivos y Uveítis
coproparasitoscópicos que fueron negativos. Artropatía axial
¿Cuál es el diagnóstico más probable en este paciente? Penfigoide ampolloso
El penfigoide ampolloso no es una entidad asociada a la enfermedad
Colitis ulcerativa inflamatoria intestinal.
Ileítis por Y. enterocolítica
Enfermedad celíaca CASO CLÍNICO 7
Amebiasis intestinal Mujer de 42 años de edad con diagnóstico reciente de
Enfermedad de Crohn hipercolesterolemia. Su último perfil de lípidos reportó
un colesterol total de 330 mg/dL. Inició hace 6 meses Esfinterotomía endoscópica
con fatiga y prurito el cuál en ocasiones es El ácido ursodesoxicólico es el tratamiento de elección en los pacientes con
colangitis biliar primaria y los pacientes que responden bioquímicamente
incapacitante. Su esposo refiere que ha notado cambio tienen mejor pronóstico que aquellos sin respuesta.
en la coloración de su piel. ¿Cuál de los siguientes tratamientos NO es útil en el
De acuerdo a los datos comentados, ¿Cuál de las manejo del prurito?
siguientes manifestaciones clínicas no sería Colestiramina
concordante con el cuadro de la paciente? Plasmaféresis
Xantelasmas Sertralina
Xantomas Rifampicina
Tiroiditis Prednisona
Boca seca Los esteroides no son útiles en el tratamiento del prurito por CBP. La primera
Colangitis de repetición elección para el tratamiento del prurito en CBP es la colestiramina. El resto
de opciones puede utilizarse en algún punto del tratamiento. La
Se trata de un cuadro típico de colangitis biliar primaria. Dicha entidad se
plasmaféresis puede ser útil para el prurito refractario a tratamiento médico
asocia con dislipidemia con hipercolesterolemia con xantomas y xantelasmas
y en casos extremos el prurito per se es una indicación de trasplante de
resultantes. La hepatopatía autoinmune asociada a colangitis de repetición
hígado.
es la colangitis esclerosante primaria.
Se realizaron exámenes de laboratorio que mostraron
BT 1 mg/dL, ALT 35 U/L, AST 33 U/L, FA 525 U/L, CLINICAL CASE 8
albúmina 4 g/dL. El USG hepático fue reportado A 60 year old woman with a history of longstanding HCV
normal. De acuerdo a su primera sospecha diagnóstica compensated cirrhosis, and a recent ankle sprain,
¿Cuál de los siguientes exámenes complementarios presents to the emergency department with a 2 week
tiene mayor probabilidad de aportar información history of abdominal swelling and lower limb edema.
diagnóstica en esta paciente? Physical examination reveals normal vital signs.
Anticuerpos anti-nucleares Abdominal examination reveals caput medusae, diffuse
Anticuerpos anti-mitocondriales abdominal distension with shifting dullness. There’s not
Anticuerpos anti-músculo liso pain upon palpation. She has never been told to have
Anticuerpos anti-LKM 1 ascites. After the consult finishes the patient
Anticuerpos anti-SLA remembered that she has been taking NSAIDS for the
El 95% de los pacientes con colangitis biliar primaria tienen anticuerpos ankle sprain for the last two weeks.
antimitocondriales positivos. Which of the following is the most useful clinical clue
El estudio previamente solicitado fue reportado for the diagnosis of ascites?
positivo. ¿Cuál es el siguiente paso en el abordaje Fluid wave test
diagnóstico? Shifting dullness
Biopsia hepática Flank percussion
Solicitar nivel de globulinas Protruding umbilical hernia
Con la información actual ya tenemos diagnóstico Iceberg sign
Anticuerpos anti-fosfolípidos Which of the following is the best next step in the
Colangiorresonancia magnética workup of this patient?
Para integrar el diagnóstico de colangitis biliar primaria basta con cumplir 2
de 3 de los siguientes criterios: Elevación de fosfatasa alcalina o GGT,
Abdominal ultrasound
Anticuerpos antimitocondriales positivos, o hallazgos histológicos típicos Diagnostic paracentesis
(lesión ductal florida). Esta paciente ya tiene 2 de los criterios y no es Abdominal CT
necesario realizar más pruebas.
Observe her response to diuretics
¿Cuál es el tratamiento de elección en este caso?
TIPS
No existe tratamiento específico The first step in the work up of any patient with ascites that has been recently
Atorvastatina diagnosed is paracentesis. The first tests to perform on the ascites samples
Prednisona are PMN count, albumin and total proteins. Once you have categorized the
ascites as portal hypertensive vs non-portal hypertensive, the further workup
Ácido Ursodesoxicólico must be individualized.
Which of the following is NOT a factor that contributes
to new onset ascites in a patient with previously CLINICAL CASE 9
compensated cirrhosis? A 40 year old obese woman without relevant medical
Acute kidney injury history, presents to emergency department with 2
Hepatocellular carcinoma hours of severe upper abdominal pain with radiation to
Portal vein thrombosis the back. She reports that this is the worst pain ever and
Progression of the primary liver disease compares it to a knife that penetrates her abdomen
Encephalopathy towards her back. Since the beginning of her symptoms
Encephalopathy, as ascites is another condition that defines decompensation she has had about 5-6 episodes of copious bilious
in cirrhosis. Encephalopathy can coexist with ascites but by itself is not a
factor that contributes to new onset ascites, rather is a marker of emesis. Her vital signs are HR 110, RR 22 and BP
decompensation. 110/70mmHg. Physical examination reveals tenderness
The ordered laboratory tests revealed Cr 1.6 mg/dL to palpation with guarding.
(two weeks ago 0.7 mg/L), FeNa 0.1. Ascites fluid Which of the following is NOT an appropriate initial
samples revealed a PMN count of 125xhpf and the option in this patient diagnosis/management?
serum ascites albumin gradient was reported in 2 Crystalloid infusion / opioid analgesia
mg/dL. Which of the following options is not Nasogastric tube
appropriate in the workup/treatment at this point? Abdominal USG
Albumin 1 g/kg Serum lipase
Hold NSAID use Begin antibiotics
Renal USG There’s no role for prophylactic antibiotics in the absence of established
infection in acute pancreatitis.
Urinary sediment
After 2 hours, with your initial management the
Furosemide
The first step in the management of acute kidney injury in cirrhosis is patient appears more comfortable, her vitals are
withholding any nephrotoxic drug. Diuretics cause azotemia primarily by stable. Laboratory tests reveal creatinine 1 mg/dL,
volume depletion. Once nephrotoxic drugs are withheld, the next step is
BUN 18 mg/dL, lipase 1200 U/L, Hematocrit 44%,
expansion of the intravascular volume with albumin (especially in those with
stage 2 or 3 acute kidney injury). Parallel, it’s important to rule out other Hemoglobin 13.5 g/dL, triglycerides 180 mg/dL,
causes of acute kidney injury as obstructive renal failure (renal USG), acute Bilirubin 5 mg/dL, Conjugated B 3.5 mg/dL,
tubular necrosis (sediment), Glomerulopathies (Sediment, erythrocyte and
protein counts in urine).
Unconjugated B 1.5 mg/dL, AST 300 UL, ALT 320 U/L,
After 48 hours of optimal medical management, the alkaline phosphatase 280 U/L. Which of the following
patient continue to deteriorate. Laboratories show Cr is the most likely cause of the clinical picture in this
3.5. You have ruled out structural causes of kidney patient?
injury and currently there’s no identified infection. Persistent obstruction of the cystic duct by a stone
Which of the following is not an appropriate option in Choledocolitiasis
the management of this patient? Pancreatic head mass
Albumin + terlipressin Pancreatic ductal calcifications
Albumin + norepinephrine Pancreas divisum
This patient clearly has pancreatitis (pain + 3x lipase elevation). The
Speed-up workup for liver transplant laboratory test abnormalities (cholestasis, moderate hypertransaminasemia)
Paracentesis as needed for tense ascites are suggestive of biliary tract obstruction.
Dialysis After 6 hours the patient deteriorates. She begins with
This patient has Type 1 hepatorrenal syndrome which is a form of acute dyspnea and tachypnea. The pulse oximetry shows a
kidney injury in cirrhotic patients characterized by peripheral vasodilation
and intense renal vasoconstriction. The treatment of choice is a combination
SaO2 of 80% (room air). Because of accessory
of a volume expander (albumin) plus vasopressors (Terlipressin or respiratory muscle use you decide to intubate the
norepinephrine) to increase renal perfusion pressures. Dialysis has no role as patient and transfer her to the ICU. Which of the
an initial treatment for hepato-renal syndrome. Hepatorrenal syndrome has
a very poor prognosis and medical treatments are only a bridge to transplant following options is NOT true about current patient
which is the most effective an also the only mean to achieve long term condition?
survival.
According to Atlanta classification she has acute severe walled off, is highly unlikely to achieve drainage and it’s better to wait until
the necrosis become walled-off to intervene. It has been described that early
pancreatitis necrosectomy increases mortality and is not an appropriate strategy.
There’s not enough information at this point to classify
the severity of this patient pancreatitis
Diagnosis of acute severe pancreatitis mainly depends
on persistent organ failure CLINICAL CASE 10
If the patient improves in the next hours and can be A 70 year old, previously healthy man presents with
extubated, the pancreatitis could be classified as painless jaundice and itching over the last 2 months. He
moderate reports a 10 kg weight loss. Physical examination reveals
The patient has some predictors of severe acute a thin man. Abdomen is soft but tender to deep
pancreatitis but right now it cannot be classified as such palpation in the epigastric area. Laboratory tests report
Atlanta classification defines acute severe pancreatitis as persistent organ
failure that lasts more than 48 hours according to Marshall Criteria. Although
a Total bilirubin of 5 mg/dL, direct bilirubin 4.1 mg/dL,
it’s very likely that this patient behaves as severe, only about 8 hours have alkaline phosphatase 220 U/L. Abdominal CT shows a
elapsed since admission. This question is to illustrate that the diagnosis of 2.8 cm solid mass localized within the pancreatic head
acute severe pancreatitis is retrospective and is one of the pitfalls of Atlanta
classification. Because of this, patient must be aggressively resuscitated. with common bile duct dilation. There’s no evidence of
After 24 hours in the ICU, the patient is improving. Her vascular invasion.
current laboratory tests reveal a total bilirubin of 3 Which of the following is the most appropriate next
mg/dL, conjugated bilirubin of 1.8 mg/dL, step in management?
unconjugated bilirubin of 1.2 mg/dL, alkaline Schedule an ERCP
phosphatase of 200 U/L, AST 150 U/L and ALT 150 U/L. Schedule a EUS with FNA
One of the senior attending physicians suggest an early Pancreaticoduodenectomy
ERCP. What’s the best strategy in the management of Refer him to medical oncology for chemotherapy
this patient biliary tract? Palliative care
The standard of care of resectable pancreatic head masses is
Program an early ERCP right now pancreatoduodenectomy. If the patient doesn’t have cholangitis, it’s
ERCP when patient is discharged from the ICU preferable not to manipulate the biliary tract before the surgery. Because
ERCP if the patient has cholangitis at some point with CT information the mass is resectable, there’s no need for further local
staging tests.
Not perform ERCP under any circumstance
The preliminary report of the surgical pathologist is a
Perform a biliary tract percutaneous drainage right now
The most powerful indication for an ERCP in the context of acute biliary
malignant lesion but he wants to further analyze the
pancreatitis is the presence of cholangitis. At this point, endoscopic drainage specimen to sign the final diagnosis. Considering
of the biliary tract may be live saving. Also in patients with persistent epidemiology, which of the following is the most likely
cholestasis ERCP is an option. Otherwise risk surpasses benefits.
diagnoses?
She is discharged to a general ward after 2 days in the
Lymphoma
ICU. After 2 weeks she appears well but begins with
Ductal adenocarcinoma
intermittent fever. Laboratory tests show a white
Acinar adenocarcinoma
blood cell count of 18000/µL. Abdominal CT shows a
IgG4 chronic pancreatitis
poorly demarcated area of peripancreatic necrosis
Retroperitoneal sarcoma
near the tail with gas bubbles. Which of the following The most frequent type of pancreatic cancer is ductal adenocarcinoma.
is the best strategy for the management at this point? Which of the following imaging features would not be
Perform a fine needle aspiration biopsy expected in a patient with your diagnosis?
Initiate antibiotics and defer interventional procedures Pancreatic duct dilation
Perform open necrosectomy Atrophic pancreas
Perform endoscopic necrosectomy Diffuse pancreatic enlargement with sausage
Percutaneous drainage appearance
This patient has an acute pancreatic necrosis (not surrounded by a capsule)
that has clear imaging signs of infection (gas bubbles) and her clinical state is Non-enhancing mass
stable. She can be safely managed with antibiotics. Because the necrosis isn’t Vascular invasion in some patients
Sausage appearance is a typical feature of Ig-G4 chronic pancreatitis.
Which of the following is NOT a typical feature of this
disease?
It’s one of the most lethal cancers
At time of presentation, tumors in the head are smaller
than those in the body and tail
Only 10-20% are resectable at diagnosis
Superior mesenteric artery invasion makes tumor
unresectable
Common liver duct invasion makes tumor unresectable
The resectability of the tumor mainly depends on vascular invasion. In
general, is worse when an artery is compromised (it could be celiac axis or
superior mesenteric artery). Sometimes there are lesions with portal vein
invasion that are amenable to surgery. When there is doubt about the
resectability status it’s better to perform an endoscopic ultrasound to clarify
this matter.
When this group of patients need medical treatment,
which of the following is NOT a usual option?
Gemcitabine
Oxaliplatin
Fluouracil
Prednisone
Irinotecan
Steroids have no role in the management of pancreatic adenocarcinoma.

Вам также может понравиться